OB Exam 4

अब Quizwiz के साथ अपने होमवर्क और परीक्षाओं को एस करें!

If nonsurgical treatment for late postpartum hemorrhage is ineffective, which surgical procedure is appropriate to correct the cause of this condition? a. Hysterectomy b. Laparoscopy c. Laparotomy d. D&C

ANS: D D&C allows examination of the uterine contents and removal of any retained placental fragments or blood clots. Hysterectomy, laparoscopy, and laparotomy are not indicated

What risk factor for peripartum depression (PPD) is likely to have the greatest effect on the woman's condition? a. Personal history of depression b. Single-mother status c. Low socioeconomic status d. Unplanned or unwanted pregnancy

ANS: A A personal history of depression is a known risk factor for peripartum depression. Being single, from a low socioeconomic status, or having an unplanned or unwanted pregnancy may contribute to depression for some women but are not strong predictors

The nurse is preparing a dose of naloxone for a newborn who weighs 6.9 pounds. How much naloxone does the nurse administer? mg

ANS: 0.31 mg The dose is 0.1 mg/kg for this 3.1-kg baby.

The provider orders an infusion of magnesium sulfate to run at 4 g/hour. The pharmacy delivers a bag of 4 g magnesium sulfate in 250 mL. At what rate does the nurse set the pump?

ANS: 250 mL/hour 4 g/250 mL = 0.16 mg/mL 4/0.016 - 250 mL/hour

Which factor is most likely to result in fetal hypoxia during a dysfunctional labor? a. Incomplete uterine relaxation b. Maternal fatigue and exhaustion c. Maternal sedation with narcotics d. Administration of tocolytic drugs

ANS: A A high uterine resting tone, with inadequate relaxation between contractions, reduces maternal blood flow to the placenta and decreases fetal oxygen supply. Maternal fatigue or sedation does not decrease uterine blood flow. Tocolytic drugs decrease contractions. This will increase uterine blood flow

A woman in preterm labor at 30 weeks of gestation receives two 12-mg doses of betamethasone intramuscularly. The purpose of this pharmacologic treatment is to a. stimulate fetal surfactant production. b. reduce maternal and fetal tachycardia associated with ritodrine administration. c. suppress uterine contractions. d. maintain adequate maternal respiratory effort and ventilation during magnesium sulfate therapy.

ANS: A Antenatal glucocorticoids given as intramuscular injections to the mother accelerate fetal lung maturity. Inderal would be given to reduce the effects of ritodrine administration. Betamethasone has no effect on uterine contractions. Calcium gluconate would be given to reverse the respiratory depressive effects of magnesium sulfate therapy.

A student nurse is preparing to administer a dose of betamethasone. What action by the student warrants intervention by the registered nurse? a. Starts a separate IV line to infuse the medication b. Tells the woman her blood glucose will be monitored more often c. Prepares an IM injection choosing a ´ needle d. Listens to the woman's lungs prior to administering the medication

ANS: A Betamethasone is given in two IM injections with the appropriate needle. When the student begins to insert a dedicated line for administering it, the nurse intervenes to stop this incorrect action. Since this drug is a steroid, blood glucose readings can rise, so diabetic patients will have more frequent blood sugars. Pulmonary edema is uncommon, but the astute nurse (or student) will listen to lung sounds prior to administration for a baseline

Birth for the nulliparous woman with a fetus in a breech presentation is usually by a. cesarean delivery. b. vaginal delivery. c. forceps-assisted delivery. d. vacuum extraction.

ANS: A Delivery for the nulliparous woman with a fetus in breech presentation is almost always cesarean section. The greatest fetal risk in the vaginal delivery of breech presentation is that the head (largest part of the fetus) is the last to be delivered. The delivery of the rest of the baby must be quick so that the infant can breathe.

Which measure may prevent mastitis in the breastfeeding mother? a. Initiating early and frequent feedings b. Nursing the infant for 5 minutes on each breast c. Wearing a tight-fitting bra d. Applying ice packs before feeding

ANS: A Early and frequent feedings prevent stasis of milk, which contributes to engorgement and mastitis. Five minutes does not adequately empty the breast. This will produce stasis of the milk. A firm-fitting bra will support the breast but not prevent mastitis. The breast should not be bound. Warm packs before feeding will increase the flow of milk

An infant with severe meconium aspiration syndrome (MAS) is not responding to conventional treatment. Which treatment may be necessary for this infant? a. Extracorporeal membrane oxygenation b. Respiratory support with ventilator c. Insertion of laryngoscope and suctioning of the trachea d. Insertion of an endotracheal tube

ANS: A Extracorporeal membrane oxygenation is a highly technical method that oxygenates the blood while bypassing the lungs, allowing the infant's lungs to rest and recover. The infant is most likely intubated and on a ventilator already. Laryngoscope insertion and tracheal suctioning are performed after birth before the infant takes the first breath.

The nurse is teaching the parents of a newborn who is going to receive phototherapy. What other measure does the nurse teach to help reduce the bilirubin? a. Increase the frequency of feedings. b. Increase oral intake of water between feedings. c. How to prepare the newborn for an exchange transfusion d. Wrap the infant in triple blankets to prevent cold stress during phototherapy

ANS: A Frequent feedings prevent hypoglycemia, provide protein to maintain albumin levels in the blood and promote gastrointestinal motility and removal of bilirubin in the stools. More frequent breastfeeding should be encouraged. Avoid offering water between feedings, because the infant may decrease his or her milk intake. Breast milk or formula is more effective at removing bilirubin from the intestines. Exchange transfusions are seldom necessary but may be performed when phototherapy cannot reduce high bilirubin levels quickly enough. Wrapping the infant in blankets will prevent the phototherapy from getting to the skin and being effective. The infant should be uncovered and unclothed.

Which data should alert the nurse that the neonate is postmature? a. Cracked, peeling skin b. Short, chubby arms and legs c. Presence of vernix caseosa d. Presence of lanugo

ANS: A Loss of vernix caseosa, which protects the fetal skin in utero, may leave the skin macerated and appearing cracked and peeling. Postmature infants usually have long, thin arms and legs. Vernix caseosa decreases in the postmature infant. Absence of lanugo is common in postmature infants

Which technique is least effective for the woman with persistent occiput posterior position? a. Lie supine and relax. b. Sit or kneel, leaning forward with support. c. Rock the pelvis back and forth while on hands and knees. d. Squat.

ANS: A Lying supine increases the discomfort of "back labor." A sitting or kneeling position may help the fetal head to rotate to occiput anterior. Rocking the pelvis encourages rotation from occiput posterior to occiput anterior. Squatting aids both rotation and fetal descent.

The nurse should expect medical intervention for subinvolution to include a. oral methylergonovine maleate (Methergine) for 48 hours. b. oxytocin intravenous infusion for 8 hours. c. oral fluids to 3000 mL/day. d. intravenous fluid and blood replacement.

ANS: A Methergine provides long-sustained contraction of the uterus and is the usual treatment. Oxytocin and oral fluids are not used for this condition. There is no indication that blood loss has occurred in this situation; if it does blood replacement may be necessary.

Nursing care of the infant with neonatal abstinence syndrome should include a. Positioning the infant's crib in a quiet corner of the nursery b. Feeding the infant on a 2-hour schedule c. Placing stuffed animals and mobiles in the crib to provide visual stimulation d. Spending extra time holding and rocking the infant

ANS: A Placing the crib in a quiet corner helps avoid excessive stimulation of the infant. These infants have an increase calorie needs but poor suck and swallow coordination. Feeding should occur to meet these needs. Stimulation should be kept to a minimum

Medications used to manage postpartum hemorrhage include which of the following? (Select all that apply.) a. Oxytocin b. Methergine c. Terbutaline d. Hemabate e. Magnesium sulfate

ANS: A, B, D Pitocin, Methergine, and Hemabate are all used to manage PPH. Terbutaline and magnesium sulfate are tocolytics; relaxation of the uterus causes or worsens PPH.

The nurse is caring for a neonate undergoing phototherapy. What action does the nurse include on the infant's care plan? a. Keep the infant's eyes covered under the light. b. Keep the infant supine at all times. c. Restrict parenteral and oral fluids. d. Dress the infant in only a T-shirt and diaper.

ANS: A Retinal damage from phototherapy should be prevented by using eye shields on the infant under the light. To ensure total skin exposure, the infant's position is changed frequently. Special attention to increasing fluid intake ensures that the infant is well hydrated. To ensure total skin exposure, the infant is not dressed.

The perinatal nurse caring for the postpartum woman understands that late postpartum hemorrhage is most likely caused by a. subinvolution of the uterus. b. defective vascularity of the decidua. c. cervical lacerations. d. coagulation disorders.

ANS: A The most common causes of late postpartum hemorrhage are subinvolution and retained placental fragments.

Early postpartum hemorrhage is defined as signs and symptoms of hypovolemia with which of the following descriptions of blood loss? a. Cumulative blood loss >1000 mL in the first 24 hours after the birth process. b. 750 mL in the first 24 hours after vaginal delivery c. Cumulative blood loss >1000 mL in the first 48 hours after the birth process d. 1500 mL in the first 48 hours after cesarean delivery

ANS: A The newest definition of early postpoartum hemorrhage is cumulative blood loss >1000 mL with signs of hypovolemia within the first 24 hours after the birth process. Hemorrhage after 24 hours is considered late postpartum hemorrhage

A newborn has meconium aspiration at birth. The nurse notes increasing respiratory distress. What action takes priority? a. Obtain an oxygen saturation. b. Notify the provider at once. c. Stimulate the baby to increase respirations. d. Prepare to initiate ECMO.

ANS: A This baby has a risk for, and signs of, persistent pulmonary hypertension. The nurse first checks an oxygen saturation then notifies the provider, or alternatively, gets the reading (and other assessments) while another nurse does the notification. This baby most likely has tachypnea so stimulation to increase respirations is not needed. ECMO may or may not be needed depending on whether or not other treatments work.

A nurse is caring for a preterm baby who weighs 4.8 pounds. What assessment finding indicates the baby is dehydrated? a. Urine output of 3.3 mL/hour b. Urine specific gravity of 1.001 c. Low serum sodium d. Weight gain of 43 g in one day

ANS: A This baby weighs 2.18 kg. Dehydration is noted with a urine output of <2 mL/kg/hour. A urine output of 3.3 mL is 1.5 mL/kg/hour and so indicates dehydration. The dilute urine specific gravity indicates overhydration as does the low serum sodium. The weight gain is normal (15 to 20 g/kg/day).

The perinatal nurse is caring for a woman in the immediate postbirth period. Assessment reveals that the woman is experiencing profuse bleeding. The most likely etiology for the bleeding is a. uterine atony. b. uterine inversion. c. vaginal hematoma. d. vaginal laceration.

ANS: A Uterine atony is marked hypotonia of the uterus. It is the leading cause of postpartum hemorrhage. The other situations can cause bleeding but are not the most common cause.

What action by the nurse is the most important action in preventing neonatal infection? a. Good hand hygiene b. Isolation of infected infants c. Separate gown technique d. Standard Precautions

ANS: A Virtually all controlled clinical trials have demonstrated that effective handwashing is responsible for the prevention of nosocomial infection in nursery units. The other actions do reduce risk but not nearly to the degree that good hand hygiene does

A nurse is caring for a late preterm infant. What action by the nurse is inconsistent with best practice to prevent cold stress? a. Wean the infant directly to an open crib. b. Check temperature every 3 to 4 hours. c. Encourage kangaroo care. d. Place infant on a radiant warmer

ANS: A Weaning to an open crib takes many steps and is not done directly because of the risk of cold stress. The other actions help prevent cold stress.

While caring for the postterm infant, the nurse recognizes that the fetus may have passed meconium prior to birth as a result of a. hypoxia in utero. b. NEC. c. placental insufficiency. d. rapid use of glycogen stores.

ANS: A When labor begins, poor oxygen reserves may cause fetal compromise. The fetus may pass meconium as a result of hypoxia before or during labor, increasing the risk of meconium aspiration. Meconium is not passed as a result of NEC, placental insufficiency, or rapid use of glycogen stores.

A woman who has had no prenatal care enters the labor and delivery unit in advanced labor. She has chickenpox. What action by the nurse is best? a. Place the woman in isolation. b. Give the woman immune globulin before delivery. c. Treat the woman with acyclovir. d. Administer antibiotics to the infant after birth

ANS: A Women with varicella infections (chickenpox or shingles) need to be in isolation (airborne and contact per the CDC). There might not be enough time to administer immune globulin to the mother before delivery, but it could be given to the baby. Acyclovir is the drug of choice for treatment, but the staff needs to be protected from this infection through isolation precautions. Antibiotics are not used for this disease

A home health care nurse is checking on a new mother with signs of obsessive-compulsive disorder. What assessment findings correlate with this condition? (Select all that apply.) a. Frequently checking on the baby b. Fear of being alone with the baby c. Woman states she feels worthless d. Woman has bought $5,000 worth of toys e. Mother states birth was very traumatic

ANS: A, B Postpartum OCD often manifests with women performing obsessive behaviors and voicing fear of being left alone with their baby. Feeling worthless is a sign of depression. A spending spree might be a sign of the manic phase of bipolar disease. Viewing the birth as traumatic may lead to PTSD

What actions can the labor and delivery nurse take to decrease a woman's chance of contracting a puerperal infection? (Select all that apply.) a. Avoid straight catheterizing the woman unless she cannot void. b. Keep vaginal examinations to a minimum. c. Change wet peripads and linens frequently. d. Maintain the woman on bedrest while laboring. e. Use good hand hygiene before and after contact with the woman.

ANS: A, B, C, E Risk for infection increases with catheterization, vaginal examinations, exposure to wet linens and pads, and poor hand hygiene. Remaining on bedrest does not reduce the chance for infection

The nurse explain to the student that which of the following factors increase a woman's risk for thrombosis? (Select all that apply.) a. Use of stirrups for a prolonged period of time b. Prolonged bedrest during or after labor and delivery c. Adherence to a strict vegetarian diet d. Excessive sweating during labor e. Maternal age greater than 30 years of age

ANS: A, B, D, E Use of stirrups for a prolonged period of time, bedrest, excessive sweating (leading to dehydration) all increase the risk of thrombosis. Vegetarian diets are not related. Maternal age >35 increases the risk

What are the priority nursing assessments for a woman receiving tocolytic therapy with terbutaline? (Select all that apply.) a. Fetal heart rate b. Maternal heart rate c. Intake and output d. Maternal blood glucose e. Maternal blood pressure f. Odor of amniotic fluid

ANS: A, B, E All assessments are important, but those most relevant to the medication include the fetal heart rate and maternal pulse, which tend to increase, and the maternal blood pressure, which tends to exhibit a wide pulse pressure. The other assessments are important but not related to this medication.

The nurse tells the nursing student that late preterm infants are at increased risk for which of the following problems? (Select all that apply.) a. Problems with thermoregulation b. Cardiac distress c. Hyperbilirubinemia d. Sepsis e. Hyperglycemia

ANS: A, C, D Problems with thermoregulation, hyperbilirubinemia, and sepsis are common with late preterm infants. They typically have respiratory distress and hypoglycemia.

A woman reports a sudden gush of fluid from her vagina and is worried about premature rupture of her membranes. What other causes of this does the nurse assess for? (Select all that apply.) a. Urinary incontinence b. Leaking of amniotic fluid c. Loss of mucous plug d. An increase in vaginal discharge e. Bloody show

ANS: A, C, D, E Urinary incontinence, loss of the mucous plug (leading to bloody show), and increased vaginal discharge can all be mistaken for PROM. Leaking amniotic fluid is an indication of PROM.

The nursing faculty explains to students on the labor and delivery unit that late preterm and term births are very different. What distinguishes the late preterm birth from a term birth? (Select all that apply.) a. Late preterm births are between 34 and 36 completed weeks of pregnancy. b. There is no real difference in mortality between the two types of births. c. Late preterm infants may appear to be full term at delivery. d. A late preterm infant who appears full term is classified full term. e. Late preterm infants need careful assessments of gestational age

ANS: A, C, E Late preterm and term deliveries are very different, with late preterm occurring between 34 and 36 completed weeks of gestation. Mortality for late preterm babies is three times higher than for term babies. Because infant appearance can be deceiving, very careful assessment are needed; the late preterm baby can appear as if he or she is full term.

A postpartum patient is at increased risk for postpartum hemorrhage if she delivers a(n) a. 5-lb, 2-oz infant with outlet forceps. b. 6.5-lb infant after a 2-hour labor. c. 7-lb infant after an 8-hour labor. d. 8-lb infant after a 12-hour labor.

ANS: B A rapid (precipitous) labor and delivery may cause exhaustion of the uterine muscle and prevent contraction. The use of forceps may cause lacerations that could lead to bleeding, but that is not as common as hemorrhage after a precipitous labor when they are usedonly in the outlet. Eight-hour and 12-hour labors are normal in length

A multiparous woman is admitted to the postpartum unit after a rapid labor and birth of a 4000-g infant. Her fundus is boggy, lochia is heavy, and vital signs are unchanged. The nurse has the woman void and massages her fundus, but her fundus remains difficult to find, and the rubra lochia remains heavy. What action should the nurse take next? a. Continue to massage the fundus. b. Notify the provider. c. Recheck vital signs. d. Insert an indwelling urinary catheter.

ANS: B After taking these corrective actions, the nurse should contact the provider and anticipate collaborative care measures. Another nurse can assess vital signs. Since the woman just voided, an indwelling catheter is not needed.

To maintain optimal thermoregulation for the premature infant, what action by the nurse is most appropriate? a. Bathe the infant once a day. b. Put an undershirt on the infant in the incubator. c. Assess the infant's hydration status. d. Lightly clothe the infant under the radiant warmer.

ANS: B Air currents around an unclothed infant will result in heat loss. Bathing causes evaporative heat loss. Assessing hydration will not maintain thermoregulation. Clothing is not worn when the infant is under a radiant warmer.

If the nurse suspects a uterine infection in the postpartum patient, she should assess the a. pulse and blood pressure. b. odor of the lochia. c. episiotomy site. d. abdomen for distention

ANS: B An abnormal odor of the lochia indicates infection in the uterus. The pulse may be altered with an infection, but the odor of the lochia will be an earlier sign and more specific. The infection may move to the episiotomy site if proper hygiene is not followed, but this does not demonstrate a uterine infection. The abdomen becomes distended usually because of a decrease of peristalsis, such as after cesarean section

What instructions should be included in the discharge teaching plan to assist the patient in recognizing early signs of complications? a. Palpate the fundus daily to ensure that it is soft. b. Notify the physician of a return to bright red bleeding. c. Report any decrease in the amount of brownish red lochia. d. The passage of clots as large as an orange can be expected.

ANS: B An increase in lochia or a return to bright red bleeding after the lochia has become pink indicates a complication. The fundus should stay firm. Large clots after discharge are a sign of complications and should be reported.

Which preterm infant should receive gavage feedings instead of a bottle? a. Sometimes gags when a feeding tube is inserted b. Is unable to coordinate sucking and swallowing c. Sucks on a pacifier during gavage feedings d. Has an axillary temperature of 98.4° F, an apical pulse of 149 beats/min, and respirations of 54 breaths/min

ANS: B An infant who cannot coordinate sucking, swallowing, and breathing should receive gavage feedings. The other infants are ready for bottle feedings.

The goal of treatment of the infant with phenylketonuria (PKU) is to a. cure cognitive delays. b. prevent central nervous system (CNS) damage. c. prevent gastrointestinal symptoms. d. prevent the renal system damage

ANS: B CNS damage can occur as a result of toxic levels of phenylalanine. No cure exists for cognitive delays should they occur. Digestive problems are a clinical manifestation of PKU, but it is more important to prevent the CNS damage. PKU does not involve renal dysfunction.

Why is adequate hydration important when uterine activity occurs before pregnancy is at term? a. Fluid and electrolyte imbalance can interfere with the activity of the uterine pacemakers. b. Dehydration may contribute to uterine irritability for some women. c. Dehydration decreases circulating blood volume, which leads to uterine ischemia. d. Fluid needs are increased because of increased metabolic activity occurring during contractions

ANS: B Dehydration can contribute to uterine irritability for some women, especially if the woman has an infection. Fluid and electrolyte imbalances are not associated with preterm labor. The woman has an increased blood volume during pregnancy. Fluid needs do not increase due to contractions.

An hour after her membranes ruptured, a laboring woman has a temperature of 38.2° C (100.7° F). What action does the nurse perform first? a. Provide cool, wet washcloths for the woman's forehead. b. Assess and document the fetal heart rate. c. Administer acetaminophen orally. d. Encourage the woman to drink clear fluids.

ANS: B Fetal tachycardia is associated with maternal fever. While all options are reasonable, the nurse needs to assess fetal well-being first.

A woman delivered a 9-lb, 10-oz baby 1 hour ago. When you arrive to perform her 15-minute assessment, she tells you that she "feels all wet underneath." You discover that both pads are completely saturated and that she is lying in a 6-inch-diameter puddle of blood. What is your first action? a. Call for help b. Assess the fundus for firmness. c. Take her blood pressure. d. Check the perineum for lacerations.

ANS: B Firmness of the uterus is necessary to control bleeding from the placental site. The nurse should first assess for firmness and massage the fundus as indicated. Calling for help is not needed unless corrective action does not improve the situation. Another nurse can take the blood pressure or the original nurse can do so after assessing the fundus and massaging it if needed. Checking the perineum for lacerations would be appropriate if the fundus was firm.

The nurse learns that the most common cause of pathologic hyperbilirubinemia is which of the following? a. Hepatic disease b. Hemolytic disorders in the newborn c. Postmaturity d. Congenital heart defect

ANS: B Hemolytic disorders in the newborn are the most common cause of pathologic jaundice. Hepatic damage and prematurity may be causes of pathologic hyperbilirubinemia, but they are not the most common cause. Congenital heart defect is not a common cause of pathologic hyperbilirubinemia in neonates

An infant with hypocalcemia is receiving an intravenous bolus of calcium. The infant's heart rate changes from 144 beats/minute to 62 beats/minute. What action by the nurse is best? a. Call for a stat EGG. b. Stop the infusion. c. Stimulate the infant. d. Administer magnesium.

ANS: B IV calcium can lead to bradycardia. When this infant's heart rate drops to 60 beats/minute, the nurse stops the infusion. A stat ECG is not necessary unless policy requires it or the bradycardia does not resolve. Stimulating the infant will not increase the heart rate. Magnesium infusion will also not increase the heart rate.

The nurse knows that a measure for preventing late postpartum hemorrhage is to a. administer broad-spectrum antibiotics. b. inspect the placenta after delivery. c. manually remove the placenta. d. pull on the umbilical cord to hasten the delivery of the placenta.

ANS: B If a portion of the placenta is missing, the clinician can explore the uterus, locate the missing fragments, and remove the potential cause of late postpartum hemorrhage. Broadspectrum antibiotics will be given if postpartum infection is suspected. Manual removal of the placenta increases the risk of postpartum hemorrhage. The placenta is usually delivered 5 to 30 minutes after birth of the baby without pulling on the cord. That can cause uterine inversion

The patient who is being treated for endometritis is placed in Fowler's position because it a. promotes comfort and rest. b. facilitates drainage of lochia. c. prevents spread of infection to the urinary tract. d. decreases tension on the reproductive organs

ANS: B Lochia and infectious material are eliminated by gravity drainage when the woman is placed in the Fowler's position.

Which woman is at greatest risk for early postpartum hemorrhage? a. A primiparous woman being prepared for an emergency cesarean birth for fetal distress b. A woman with severe preeclampsia on magnesium sulfate whose labor is being induced c. A multiparous woman with an 8-hour labor d. A primigravida in spontaneous labor with preterm twins

ANS: B Magnesium sulfate administration during labor poses a risk for PPH. Magnesium acts as a smooth muscle relaxant, thereby contributing to uterine relaxation and atony. The other situations do not post risk factors or causes of early PPH

A provider left an order for a woman to have Methylergonovine 0.2 mg IM. The nurse assesses the woman and finds her vital signs to be: temperature 37.9° C (100.2° F), pulse 90 beats/minute, respirations 18 breaths/minute, and blood pressure 152/90 mm Hg. What action by the nurse is most appropriate? a. Administer acetaminophen first. b. Check policy for administration. c. Give the medication as prescribed. d. Consult with the provider.

ANS: B Methylergonovine is contraindicated in women with hypertension. The nurse should check the agency's policy to see at what blood pressure reading this medication should be held. After checking the policy, the nurse can consult the provider if it can't be given. Acetaminophen is not related to this situation

The difference between physiologic and nonphysiologic jaundice is that nonphysiologic jaundice a. usually results in kernicterus. b. appears during the first 24 hours of life. c. results from breakdown of excessive erythrocytes not needed after birth. d. begins on the head and progresses down the body

ANS: B Nonphysiologic jaundice appears during the first 24 hours of life, whereas physiologic jaundice appears after the first 24 hours of life. Pathologic jaundice may lead to kernicterus, but it needs to be stopped before that occurs. Both jaundices are the result of the breakdown of erythrocytes. Pathologic jaundice is due to a pathologic condition, such as Rh incompatibility

Parents of a newborn with phenylketonuria are anxious to learn about the appropriate treatment for their infant. What topic does the nurse include in the teaching plan? a. Fluid and sodium restrictions b. A phenylalanine-free diet c. Progressive mobility and splinting d. A protein-rich diet

ANS: B Phenylketonuria is treated with a special diet that restricts phenylalanine intake. Fluid and sodium restrictions are not included in this plan. Mobility and splinting are not included in the plan. A protein-rich diet is not in the plan

What is most helpful in preventing premature birth? a. High socioeconomic status b. Adequate prenatal care c. Transitional Assistance to Needy Families d. Women, Infants, and Children nutritional program

ANS: B Prenatal care is vital in identifying possible problems. Women from higher economic status are more likely to seek adequate prenatal care, but it is the care that is most helpful. Government programs help with specific needs of the pregnant woman, but adequate care is more important

In caring for the preterm infant, what complication is thought to be a result of high arterial blood oxygen level? a. Necrotizing enterocolitis (NEC) b. Retinopathy of prematurity (ROP) c. Bronchopulmonary dysplasia (BPD) d. Intraventricular hemorrhage (IVH)

ANS: B ROP is thought to occur as a result of high levels of oxygen in the blood. NEC is due to the interference of blood supply to the intestinal mucosa. Necrotic lesions occur at that site. BPD is caused by the use of positive pressure ventilation against the immature lung tissue. IVH is due to rupture of the fragile blood vessels in the ventricles of the brain. It is most often associated with hypoxic injury, increased blood pressure, and fluctuating cerebral blood flow.

An important independent nursing action to promote normal progress in labor is a. assessing the fetus. b. encouraging urination about every 1 to 2 hours. c. allowing the woman to stay in her preferred position. d. regulating intravenous fluids.

ANS: B The bladder can reduce room in the woman's pelvis that is needed for fetal descent and can increase her discomfort. Assessment of the fetus is an important task, but will not promote normal progression of labor. Position changes help labor progress and should be encouraged. Maintaining hydration is an important task, but it will not promote normal progression of labor.

A woman in labor at 34 weeks of gestation is hospitalized and treated with intravenous magnesium sulfate for 18 to 20 hours. When the magnesium sulfate is discontinued, which oral drug will probably be prescribed for continuation of the tocolytic effect? a. Ritodrine b. Terbutaline c. Calcium gluconate d. Pitocin

ANS: B The woman receiving decreasing doses of magnesium sulfate is often switched to oral terbutaline to maintain tocolysis for 48 hours. The terbutaline will probably be discontinued prior to discharge. Ritodrine is the only drug approved by the FDA for tocolysis; however, it is rarely used because of significant side effects. Calcium gluconate reverses magnesium sulfate toxicity. The drug should be available for complications of magnesium sulfate therapy. Pitocin is used to augment labor, not stop it

A nurse is caring for a preterm infant who has a weak cry and is irritable. What action by the nurse is best? a. Assess the infant for pain. b. Take the infant's temperature. c. Obtain a bedside glucose reading. d. Reduce stimulation in the environment.

ANS: B These are signs of inadequate thermoregulation. The nurse should assess the infant's temperature first. The other actions do not address thermoregulation

A neonate has white patches in her mouth that bled when the mother tried wiping them away. What action by the nurse is best? a. Tell the mother to leave the patches alone. b. Assess the mother for a perineal rash. c. Give the infant medicated pacifiers. d. Test the infant for toxoplasmosis.

ANS: B These patches are characteristic of maternal infection with candidiasis or yeast. The nurse assesses the mother's perineal area for a rash. Telling the mother to leave the rash alone may be appropriate information but does not get to the bottom of the issue. The nurse should not provide medication without knowing what is being treated. The baby does not have toxoplasmosis.

The nursing student observes a laboring woman doing lunges to the left side and asks for an explanation of this activity. What response by the nurse is best? a. It decreases the pain associated with back labor. b. It promotes rotation of the fetal occiput to an anterior position. c. It relieves the cramping associated with a prolonged labor. d. It causes the pelvic inlet to open wider in preparation for birth

ANS: B This action encourages rotation of the fetal head to the anterior position. It does relieve back labor, but this response does not explain why. It does not relieve cramping or open the pelvic inlet

A premature infant never seems to sleep longer than an hour at a time. Each time a light is turned on, an incubator closes, or people talk near her crib, she wakes up and cries inconsolably until held. The correct nursing diagnosis is ineffective coping related to a. severe immaturity. b. environmental stress. c. physiologic distress. d. behavioral responses.

ANS: B This nursing diagnosis is the most appropriate for this infant. Light and sound are known adverse stimuli that add to an already stressed premature infant. The nurse must monitor the environment closely for sources of overstimulation. The other diagnoses do not recognize that fact

A steady trickle of bright red blood from the vagina in the presence of a firm fundus suggests a. uterine atony. b. lacerations of the genital tract. c. perineal hematoma. d. infection of the uterus.

ANS: B Undetected lacerations will bleed slowly and continuously. Bleeding from lacerations is uncontrolled by uterine contraction. The fundus is not firm with uterine atony. A hematoma would be internal. Swelling and discoloration would be noticed, but bright bleeding would not be. With an infection of the uterus there would be an odor to the lochia and systemic symptoms such as fever and malaise.

A woman who has a history of frequent substance abuse is close to delivering. What action by the nurse is best? a. Notify social services of the situation prior to the birth. b. Draw up and label a syringe of naloxone. c. Administer naloxone if the baby shows signs of withdrawal. d. Prepare to administer naloxone to the mother.

ANS: B When anticipating the delivery of a baby whose mother is addicted to opioids, the nurse prepares to give the newborn naloxone for respiratory depression. To administer the drug in the fastest way possible, the nurse prepares a syringe with the medication. Then when the baby's weight is known, the nurse discards the excess drug and administers the correct dose to the baby. Social services will need to be involved but not at this point; the medication is the priority. The naloxone may cause signs of withdrawal in the infant. The baby gets the naloxone, not the mother.

The causes of preterm labor are not fully understood although many factors have been associated with early labor. These include (Select all that apply.) a. Singleton pregnancy b. History of cone biopsy c. Smoking d. Short cervical length e. Higher level of education

ANS: B, C, D A history of cone biopsies, smoking, and a short cervical length are all associated with early labor. Singleton pregnancy and higher level of education are not.

Newborns whose mothers are substance abusers frequently have what behaviors? (Select all that apply.) a. Circumoral cyanosis b. Decreased amounts of sleep c. Hyperactive Moro (startle) reflex d. Difficulty feeding e. Weak cry

ANS: B, C, D The infant exposed to drugs in utero often has poor sleeping patterns, hyperactive reflexes, and uncoordinated sucking and swallowing behavior. They do not have circumoral cyanosis and will have a high-pitched cry

Some infants develop hypoxic-ischemic encephalopathy after asphyxia. Therapeutic hypothermia has been used to improve neurologic outcomes for these infants. Criteria for the use of this modality include (Select all that apply.) a. The infant must be 28 weeks gestation or greater. b. Have evidence of an acute hypoxic event. c. Be in a facility they can initiate treatment within 6 hours. d. The infant must be 36 or more weeks' gestation. e. The treatment must be initiated within the first 12 hours of life.

ANS: B, C, D The infant must be at least 36 weeks of gestation to meet the criteria for therapeutic hypothermia. Treatment should be initiated within the first 6 hours of life, ideally at a tertiary care center. The infant must have evidence of perinatal hypoxic-ischemic episodes

An important nursing factor during the care of the infant in the NICU is assessment for signs of adequate parental attachment. The nurse must observe for signs that bonding is not occurring as expected. These include (Select all that apply.) a. using positive terms to describe the infant. b. showing interest in other infants equal to that of their own. c. naming the infant. d. decreasing the number and length of visits. e. refusing offers to hold and care for the infant

ANS: B, D, E Bonding is not progressing as expected when parents show interest in other babies equal to that of their own, decreasing the number and length of visits, and refusing to hold and help care for the infant. Using positive terms to describe the baby and naming the infant are signs that bonding is occurring

A woman just received an injection of carboprost, 2500 mcg IM. What actions by the nurse take priority? (Select all that apply.) a. Assess for nausea and vomiting b. Assess fetal well-being. c. Administer acetaminophen for headache. d. Monitor urine output. e. Notify the provider immediately.

ANS: B, E The usual dose of carboprost is 250 mcg, so this excessive dose could lead to uterine rupture. The nurse monitors the woman for signs of this and continually monitors the fetus for well-being. The provider would be notified and agency policy followed for variance reporting. Nausea, vomiting, and headache are side effects of the usual dose of the drug. This drug is excreted through urine, so monitoring urine output is important but not as critical as checking fetal well-being and notifying the provider

The nurse should suspect uterine rupture if a. fetal tachycardia occurs. b. the woman becomes dyspneic. c. contractions abruptly stop during labor. d. labor progresses unusually quickly.

ANS: C A large rupture of the uterus will disrupt its ability to contract. Fetal tachycardia is a sign of hypoxia. Dyspnea and unusually quick labor are not signs of rupture.

The nurse present at the delivery is reporting to the nurse who will be caring for the neonate after birth. What information might be included for an infant who had thick meconium in the amniotic fluid? a. The infant had Apgar scores of 6 and 8. b. An IV was started immediately after birth to treat dehydration c. No meconium was found below the vocal cords when they were examined. d. The parents spent an hour bonding with the baby after birth.

ANS: C A laryngoscope is inserted to examine the vocal cords. If no meconium is below the cords, probably no meconium is present in the lower air passages, and the infant will not develop meconium aspiration syndrome. Apgar scores are important but not directly related to meconium. There is no relationship between dehydration and meconium fluid. Bonding is an expected occurrence

A woman who is 32 weeks pregnant telephones the nurse at her obstetrician's office and complains of constant backache. She asks what pain reliever is safe for her to take. The best nursing response is a. "Back pain is common at this time during pregnancy due to poor posture." b. "Acetaminophen is acceptable during pregnancy; however, do not take aspirin." c. "You should come into the office and let the doctor check you." d. "Try a warm bath or using a heating pad."

ANS: C A prolonged backache is one of the subtle symptoms of preterm labor. Early intervention may prevent preterm birth. The woman should be assessed before trying any home care measures

Nursing measures that help prevent postpartum urinary tract infection include which of the following? a. Promoting bed rest for 12 hours after delivery b. Discouraging voiding until the sensation of a full bladder is present c. Forcing fluids to at least 3000 mL/day d. Encouraging the intake of orange, grapefruit, or apple juice

ANS: C Adequate fluid intake of 2500 to 3000 mL/day prevents urinary stasis, dilutes urine, and flushes out waste products. The woman should be encouraged to ambulate early. With pain medications, trauma to the area, and anesthesia, the sensation of a full bladder may be decreased. The woman needs to be encouraged to void frequently. Juices such as cranberry juice can discourage bacterial growth

After a birth complicated by a shoulder dystocia, what action by the nurse is most appropriate? a. Give supplemental oxygen with a small face mask. b. Encourage the parents to hold the infant. c. Palpate the infant's clavicles. d. Perform a complete newborn assessment.

ANS: C Because of the shoulder dystocia, the infant's clavicles may have been fractured. Palpation is a simple assessment to identify crepitus or deformity that requires follow-up. There is no indication for oxygen. The infant needs to be assessed for clavicle fractures before excessive movement. A complete newborn assessment is necessary for all newborns, but assessment of the clavicle is top priority for this infant

What factor found in maternal history should alert the nurse to the potential for a prolapsed umbilical cord? a. Oligohydramnios b. Pregnancy at 38 weeks of gestation c. Presenting part at station -3 d. Meconium-stained amniotic fluid

ANS: C Because the fetal presenting part is positioned high in the pelvis and is not well applied to the cervix, a prolapsed cord could occur if the membranes rupture. Hydramnios puts the woman at high risk for a prolapsed umbilical cord. A very small fetus, normally preterm, puts the woman at risk for a prolapsed umbilical cord. Meconium-stained amniotic fluid shows that the fetus already has been compromised, but it does not increase the chance of a prolapsed cord.

Transient tachypnea of the newborn (TTN) is thought to occur as a result of a. a lack of surfactant. b. hypoinflation of the lungs. c. delayed absorption of fetal lung fluid. d. a slow vaginal delivery associated with meconium-stained fluid.

ANS: C Delayed absorption of fetal lung fluid is thought to be the reason for TTN. Lack of surfactant and hypoinflation of the lungs are not related to TTN. A slow vaginal delivery will help prevent TTN

Of all the signs seen in infants with respiratory distress syndrome, which sign is especially indicative of the syndrome? a. Pulse more than 160 beats/min b. Circumoral cyanosis c. Grunting d. Substernal retractions

ANS: C Grunting increases the pressure inside the alveoli to keep them open when surfactant is insufficient. This is a characteristic and often early sign of RDS. The other assessments are not specific to RDS.

With regard to eventual discharge of the high-risk newborn or transfer to a different facility, nurses and families should be aware that a. infants will stay in the NICU until they are ready to go home. b. once discharged to home, the high-risk infant should be treated like any healthy term newborn. c. parents of high-risk infants need special support and detailed contact information. d. if a high-risk infant and mother need transfer to a specialized regional center, it is better to wait until after birth and the infant is stabilized

ANS: C High-risk infants can cause profound parental stress and emotional turmoil. Parents need support, special teaching, and quick access to various resources available to help them care for their baby. Parents and their high-risk infant should get to spend a night or two in a predischarge room, where care for the infant is provided away from the NICU. Just because high-risk infants are discharged does not mean they are normal, healthy babies. Follow-up by specialized practitioners is essential. Ideally, the mother and baby are transported with the fetus in utero; this reduces neonatal morbidity and mortality

Which statement is true about large for gestational age (LGA) infants? a. They weigh more than 3500 g. b. They are above the 80th percentile on gestational growth charts. c. They are prone to hypoglycemia, polycythemia, and birth injuries. d. Postmaturity syndrome and fractured clavicles are the most common complications

ANS: C Hypoglycemia, polycythemia, and birth injuries are common in LGA infants. LGA infants are determined by their weight compared to their age. They are above the 90th percentile on the gestational growth charts. Birth injuries are a problem, but postmaturity syndrome is not an expected complication with LGA infants.

The maternity nurse knows that which disorder can be triggered by a birth the woman views as traumatic? a. A phobia b. Panic disorder c. Posttraumatic stress disorder (PTSD) d. Obsessive-compulsive disorder (OCD)

ANS: C In PTSD, women perceive childbirth as a traumatic event. They have nightmares and flashbacks about the event, anxiety, and avoidance of reminders of the traumatic event. This will not lead to phobias, panic disorder, or OCD

A nurse is assessing an SGA infant with asymmetric intrauterine growth restriction. What assessment finding correlates with this condition? a. One side of the body appears slightly smaller than the other. b. All body parts appear proportionate. c. The head seems large compared with the rest of the body. d. The extremities are disproportionate to the trunk.

ANS: C In asymmetric intrauterine growth restriction, the head is normal in size but appears large because the infant's body is long and thin due to lack of subcutaneous fat. The left and right side growth should be symmetric. With asymmetric intrauterine growth restrictions, the body appears smaller than normal compared to the head. The body parts are out of proportion, with the body looking smaller than expected due to the lack of subcutaneous fat. The body, arms, and legs have lost subcutaneous fat so they will look small compared to the head

Which nursing measure is appropriate to prevent thrombophlebitis in the recovery period after a cesarean birth? a. Roll a bath blanket and place it firmly behind the knees. b. Limit oral intake of fluids for the first 24 hours. c. Assist the patient in performing gentle leg exercises. d. Ambulate the patient as soon as her vital signs are stable.

ANS: C Leg exercises and passive range of motion promote venous blood flow and prevent venous stasis while the patient is still on bed rest. The blanket behind the knees will cause pressure and decrease venous blood flow. Limiting oral intake will produce hemoconcentration, which may lead to thrombophlebitis. The patient may not have full return of leg movements, and ambulating is contraindicated until she has full motion and sensation.

Which patient situation presents the greatest risk for the occurrence of hypotonic dysfunction during labor? a. A primigravida who is 17 years old b. A 22-year-old multiparous woman with ruptured membranes c. A multiparous woman at 39 weeks of gestation who is expecting twins d. A primigravida woman who has requested no analgesia during her labor

ANS: C Overdistention of the uterus in a multiple pregnancy is associated with hypotonic dysfunction because the stretched uterine muscle contracts poorly. A young primigravida usually will have good muscle tone in the uterus. This prevents hypotonic dysfunction. There is no indication that this woman's uterus is overdistended, which is the main cause of hypotonic dysfunction. A primigravida usually will have good uterine muscle tone, and there is no indication of an overdistended uterus

To provide adequate postpartum care, the nurse should be aware that peripartum depression (PPD) a. is the "baby blues," plus the woman has a visit with a counselor or psychologist. b. does not affect the father who can then care for the baby. c. is distinguished by pervasive sadness that lasts at least 2 weeks. d. will disappear on its own without outside help.

ANS: C PPD is characterized by a persistent depressed state. The woman is unable to feel pleasure or love although she is able to care for her infant. She often experiences generalized fatigue, irritability, little interest in food and sleep disorders. PPD is more serious and persistent than postpartum baby blues. Fathers are often affected. Most women need professional help to get through PPD, including pharmacologic intervention

Overstimulation may cause increased oxygen use in a preterm infant. Which nursing intervention helps to avoid this problem? a. Group all care activities together to provide long periods of rest. b. While giving your report to the next nurse, stand in front of the incubator and talk softly about how the infant responds to stimulation. c. Teach the parents signs of overstimulation, such as turning the face away or stiffening and extending the extremities and fingers. d. Keep charts on top of the incubator so the nurses can write on them there.

ANS: C Parents should be taught these signs of overstimulation so they will learn to adapt their care to the needs of their infant. This may understimulate the infant during those long periods and overtire the infant during the procedures. Talking in front of the incubator could overstimulate the baby. Placing objects on top of the incubator or using it as a writing surface increases the noise inside.

A preterm infant is on a respirator with intravenous lines and much equipment around her when her parents come to visit for the first time. What action by the nurse is most important? a. Suggest that the parents visit for only a short time to reduce their anxieties. b. Reassure the parents that the baby is progressing well. c. Encourage the parents to touch her. d. Discuss the care they will give her when she goes home.

ANS: C Physical contact with the infant is important to establish early bonding. The nurse as the support person and teacher is responsible for shaping the environment and making the care giving responsive to the needs of both the parents and the infant. The nurse should encourage the parents to touch their baby and show them how to do so safely. Bonding needs to occur, and this can be fostered by encouraging the parents to spend time with the infant. It is important to keep the parents informed about the infant's progression, but the nurse needs to be honest with the explanations. Discussing home care needs to wait until the parents are ready and discharge is closer with known needs.

In planning for home care of a woman with preterm labor, the nurse needs to address which concern? a. Nursing assessments will be different from those done in the hospital setting. b. Restricted activity and medications will be necessary to prevent recurrence of preterm labor. c. Prolonged bed rest may cause negative physiologic effects. d. Home health care providers will be necessary

ANS: C Prolonged bed rest may cause adverse effects such as weight loss, loss of appetite, muscle wasting, weakness, bone demineralization, decreased cardiac output, risk for thrombophlebitis, alteration in bowel functions, sleep disturbance, and prolonged postpartum recovery. Nursing assessments will differ somewhat from those performed in the acute care setting, but this is not the concern that needs to be addressed. Restricted activity and medication may prevent preterm labor but not in all women. Many, but not all, women will receive home health nurse visits.

Decreased surfactant production in the preterm lung is a problem because surfactant a. causes increased permeability of the alveoli. b. provides transportation for oxygen to enter the blood supply. c. keeps the alveoli open during expiration. d. dilates the bronchioles, decreasing airway resistance.

ANS: C Surfactant prevents the alveoli from collapsing each time the infant exhales, thus reducing the work of breathing. It does not cause increased permeability, provide transportation of oxygen or dilate the bronchioles.

The mother-baby nurse must be able to recognize what sign of thrombophlebitis? a. Visible varicose veins b. Positive Homans sign c. Local tenderness, heat, and swelling d. Pedal edema in the affected leg

ANS: C Tenderness, heat, and swelling are classic signs of thrombophlebitis that appear at the site of the inflammation. Varicose veins may predispose the woman to thrombophlebitis but are not a sign. A positive Homans sign may be caused by a strained muscle or contusion. Edema may be caused by other factors, and the edema with thrombophlebitis may be more extensive. Edema may be more involved than pedal.

What action does the nurse add to the plan of care for an infant experiencing symptoms of drug withdrawal? a. Keeping the newborn sedated b. Feeding every 4 to 6 hours to allow extra rest c. Swaddling the infant snugly d. Playing soft music during feeding

ANS: C The infant should be wrapped snugly to reduce self-stimulation behaviors and protect the skin from abrasions. The baby is not kept sedated. The infant should be fed in small, frequent amounts and burped well to diminish aspiration and maintain hydration. The infant should not be stimulated (such as with music), because this will increase activity and potentially increase CNS irritability.

A mother with mastitis is concerned about breastfeeding while she has an active infection. The nurse should explain that a. the infant is protected from infection by immunoglobulins in the breast milk. b. the infant is not susceptible to the organisms that cause mastitis. c. the organisms that cause mastitis are not passed to the milk. d. the organisms will be inactivated by gastric acid

ANS: C The organisms are localized in the breast tissue and are not excreted in the breast milk. The mother is just producing the immunoglobulin from this infection, so it is not available for the infant. Because of an immature immune system, infants are susceptible to many infections. However, this infection is in the breast tissue and is not excreted in the breast milk. The organism will not get into the infant's gastrointestinal system.

The nurse is observing a parent holding a preterm infant. The infant is sneezing, yawning, and extending the arms and legs. What action by the nurse is best? a. Cover the infant with a warmed blanket. b. Encourage the parent to do kangaroo care. c. Encourage the parent to place the infant back in the warmer d. Have the parent fold the infant's arms across the chest.

ANS: C These are signs that the preterm infant is overstimulated. The parent should place the infant back in her warmer, and the nurse can turn down the lights and limit noise. The other suggestions will not help decrease stimulation

Near the end of the first week of life, an infant who has not been treated for any infection develops a copper-colored, maculopapular rash on the palms and around the mouth and anus. The newborn is showing signs of a. gonorrhea. b. herpes simplex virus infection. c. congenital syphilis. d. HIV.

ANS: C This rash is indicative of congenital syphilis. The lesions may extend over the trunk and extremities. This is not characteristic of gonorrhea, herpes, or HIV.

When a woman is diagnosed with postpartum psychosis, one of the main concerns is that she may a. have outbursts of anger. b. neglect her hygiene. c. harm her infant. d. lose interest in her husband.

ANS: C Thoughts of harm to one's self or the infant are among the most serious symptoms of PPD and require immediate assessment and intervention. The other problems can be attributed to postpartum psychosis, but the major concern is harm to the infant.

Providing care for the neonate born to a mother who abuses substances can present a challenge for the health care team. Nursing care for this infant requires a multisystem approach. The first step in the provision of this care is a. pharmacologic treatment. b. reduction of environmental stimuli. c. neonatal abstinence syndrome scoring. d. adequate nutrition and maintenance of fluid and electrolyte balance

ANS: C Various scoring systems exist to determine the number, frequency, and severity of behaviors that indicate neonatal abstinence syndrome. The score is helpful in determining the necessity of drug therapy to alleviate withdrawal. Pharmacologic treatment is based on the severity of withdrawal symptoms. Swaddling, holding, and reducing environmental stimuli are essential in providing care to the infant who is experiencing withdrawal. However, the scoring helps provide definitive care. Fluids and electrolyte balance are appropriate for any infant.

A woman is having her first child. She has been in labor for 15 hours. Two hours ago, her vaginal examination revealed the cervix to be dilated to 5 cm and 100% effaced, and the presenting part was at station 0. Five minutes ago, her vaginal examination indicated that there had been no change. What abnormal labor pattern is associated with this description? a. Prolonged latent phase b. Protracted active phase c. Secondary arrest d. Protracted descent

ANS: C With a secondary arrest of the active phase, the progress of labor has stopped. This patient has not had any anticipated cervical change, indicating an arrest of labor. Dilation at 5 cm is past the latent phase. This does not describe a "protracted" labor.

A primigravida has just delivered a healthy infant girl. The nurse is about to administer erythromycin ointment in the infant's eyes when the mother asks, "What is that medicine for?" The nurse responds a. "It is an eye ointment to help your baby see you better." b. "It is to protect your baby from contracting herpes from your vaginal tract." c. "Erythromycin is given to prevent a gonorrheal infection." d. "This medicine will protect your baby's eyes from drying out."

ANS: C With the prophylactic use of erythromycin, the incidence of gonococcal conjunctivitis has declined to less than 0.5%. Eye prophylaxis is administered at or shortly after birth to prevent ophthalmia neonatorum. Erythromycin has no bearing on enhancing vision and is not used for herpes infections or lubrication.

Which combination of expressing pain could be demonstrated in a neonate? a. Low-pitched crying, tachycardia, eyelids open wide b. Cry face, flaccid limbs, closed mouth c. High-pitched, shrill cry, withdrawal, change in heart rate d. Cry face, eye squeeze, increase in blood pressure

ANS: D Cry face, eye squeeze, and an increase in blood pressure indicate pain. The other manifestations are not those of pain in the neonate

A primigravida at 40 weeks of gestation is having uterine contractions every 1.5 to 2 minutes and says that they are very painful. Her cervix is dilated 2 cm and has not changed in 3 hours. The woman is crying and wants an epidural. What is the likely status of this woman's labor? a. She is exhibiting hypotonic uterine dysfunction. b. She is experiencing a normal latent stage. c. She is exhibiting hypertonic uterine dysfunction d. She is experiencing pelvic dystocia.

ANS: C Women who experience hypertonic uterine dysfunction, or primary dysfunctional labor, often are anxious first-time mothers who are having painful and frequent contractions that are ineffective at causing cervical dilation or effacement to progress. With hypotonic uterine dysfunction, the woman initially makes normal progress into the active stage of labor and then the contractions become weak and inefficient or stop altogether. This is not a normal latent stage. Pelvic dystocia can occur whenever contractures of the pelvic diameters reduce the capacity of the bony pelvis, including the inlet, midpelvis, outlet, or any combination of these planes.

The fetus in a breech presentation is often born by cesarean delivery because a. the buttocks are much larger than the head. b. postpartum hemorrhage is more likely if the woman delivers vaginally c. internal rotation cannot occur if the fetus is breech. d. compression of the umbilical cord is more likely.

ANS: D After the fetal legs and trunk emerge from the woman's vagina, the umbilical cord can be compressed between the maternal pelvis and the fetal head if a delay occurs in the birth of the head. The head is the largest part of a fetus. There is no relationship between breech presentation and postpartum hemorrhage. Internal rotation can occur with a breech.

Which actions by the nurse may prevent infections in the labor and delivery area? a. Vaginal examinations every hour while the woman is in active labor b. Use of clean techniques for all procedures c. Cleaning secretions from the vaginal area by using back-to-front motion d. Keeping underpads and linens as dry as possible

ANS: D Bacterial growth prefers a moist, warm environment. Vaginal examinations should be limited to decrease transmission of vaginal organisms into the uterine cavity. Use an aseptic technique if membranes are not ruptured; use a sterile technique if membranes are ruptured. Vaginal drainage should be removed with a front-to-back motion to decrease fecal contamination.

The infant of a mother with diabetes is hypoglycemic. What type of feeding should be instituted first? a. Glucose water in a bottle b. D5W intravenously c. Formula via nasogastric tube d. Breast milk

ANS: D Breast milk is metabolized more slowly and provides longer normal glucose levels. Breast milk is best for nearly all babies. High levels of dextrose correct the hypoglycemia but will stimulate the production of more insulin. Oral feedings are tried first; intravenous lines should be a later choice if the hypoglycemia continues. Formula does provide longer normal glucose levels but would be administered via bottle, not by tube feeding unless the baby is unable to take oral feedings.

With regard to the care management of preterm labor, nurses should be aware that a. teaching pregnant women the symptoms probably causes more harm through false alarms. b. Braxton Hicks contractions often signal the onset of preterm labor. c. because preterm labor is likely to be the start of an extended labor, a woman with symptoms can wait several hours before contacting the primary caregiver. d. the diagnosis of preterm labor is based on gestational age, uterine activity, and progressive cervical change

ANS: D Gestational age of 20 to 37 weeks, uterine contractions, and a thinning cervix are all indications of preterm labor. It is essential that nurses teach women how to detect the early symptoms of preterm labor. Braxton Hicks contractions resemble preterm labor contractions, but they are not true labor. Waiting too long to see a health care provider could result in essential medications failing to be administered.

When caring for a postpartum woman experiencing hypovolemic shock, the nurse recognizes that the most objective and least invasive assessment of adequate organ perfusion and oxygenation is a. absence of cyanosis in the buccal mucosa. b. cool, dry skin. c. diminished restlessness. d. decreased urinary output.

ANS: D Hemorrhage may result in hypovolemic shock. Shock is an emergency situation in which the perfusion of body organs may become severely compromised, and death may occur. The presence of adequate urinary output indicates adequate tissue perfusion. The assessment of the buccal mucosa for cyanosis can be subjective in nature. The presence of cool, pale, clammy skin is an indicative finding associated with hypovolemic shock. Restlessness indicates decreased cerebral perfusion.

A nurse is participating in a neonatal resuscitation. What action by the nurse takes priority? a. Suction the mouth and nose. b. Stimulate the infant by rubbing the back. c. Perform the Apgar test. d. Place the infant in a preheated warmer.

ANS: D In a resuscitation situation, the nurse places the newborn in a preheated warmer immediately to reduce cold stress. Next position the infant in a "sniffing" position. Suctioning is the third step. Drying the infant is fourth, although if more than one health care provider is present, drying can occur simultaneously with the other actions.

A mother with diabetes has done some reading about the effects of the condition on her newborn. Which statement shows a misunderstanding that should be clarified by the nurse? a. "Although my baby is large, some women with diabetes have very small babies because the blood flow through the placenta may not be as good as it should be." b. "My baby will be watched closely for signs of low blood sugar, especially during the early days after birth." c. "The red appearance of my baby's skin is due to an excessive number of red blood cells." d. "My baby's pancreas may not produce enough insulin because the cells became smaller than normal during my pregnancy."

ANS: D Infants of diabetic mothers may have hypertrophy of the islets of Langerhans, which may cause them to produce more insulin than they need. The other statements are correctand show good understanding.

A laboring patient in the latent phase is experiencing uncoordinated, irregular contractions of low intensity. How should the nurse respond to complaints of constant cramping pain? a. "You are only 2 cm dilated, so you should rest and save your energy for when the contractions get stronger." b. "You must breathe more slowly and deeply so there is greater oxygen supply for your uterus. That will decrease the pain." c. "Let me take off the monitor belts and help you get into a more comfortable position." d. "I have notified the doctor that you are having a lot of discomfort. Let me rub your back and see if that helps."

ANS: D Intervention is needed to manage the dysfunctional pattern. Offering support and comfort is important to help the patient cope with the situation. Telling the woman to rest is belittling her complaints. Breathing will not reduce the pain. Fetal monitoring should continue as the woman changes positions

Which action should be initiated to limit hypovolemic shock when uterine inversion occurs? a. Administer oxygen at 31 L/min by nasal cannula. b. Administer an oxytocin by intravenous push. c. Monitor fetal heart rate every 5 minutes. d. Increase the intravenous infusion rate.

ANS: D Intravenous fluids are necessary to replace the lost blood volume that occurs in uterine inversion. The woman may need blood products as well. Administering oxygen will not prevent hypovolemic shock. Oxytocin should not be given until the uterus is repositioned. A uterine inversion occurs during the third stage of labor.

A woman who had two previous cesarean births is in active labor, when she suddenly complains of pain between her scapulae. The nurse's priority action is to a. reposition the woman with her hips slightly elevated. b. observe for abnormally high uterine resting tone. c. decrease the rate of nonadditive intravenous fluid. d. notify the provider promptly and prepare the woman for surgery.

ANS: D Pain between the scapulae may occur when the uterus ruptures, because blood accumulates under the diaphragm. This is an emergency that requires surgical intervention so the nurse notifies the provider and prepares the woman for surgery. Repositioning the woman with her hips slightly elevated is the treatment for a prolapsed cord. That position in this scenario would cause respiratory difficulties. Since the uterus is no longer able to contract, high resting tones cannot be assessed. However, high resting tones during labor indicate a risk for uterine rupture. The woman is now at high risk for shock. Nonadditive intravenous fluids should be increased

The nursing student learns that transmission of HIV from mother to baby occurs in which fashion? a. From the maternal circulation only in the third trimester b. From the use of unsterile instruments c. Only through the ingestion of amniotic fluid d. Through the ingestion of breast milk from an infected mother

ANS: D Postnatal transmission of HIV through breastfeeding may occur. Transplacental transmission can occur at any time during pregnancy. Unsterile instruments are possible sources of transmission but highly unlikely. Transmission of HIV may also occur during birth from blood or secretions. Transmission of HIV from the mother to the infant may occur transplacentally at various gestational ages. This is highly unlikely as most health care facilities must meet sterility standards for all instrumentation.

Which condition is a transient, self-limiting mood disorder that affects new mothers after childbirth? a. Postpartum depression b. Postpartum psychosis c. Postpartum bipolar disorder d. Postpartum blues

ANS: D Postpartum blues, or "baby blues," is a transient self-limiting disease that is believed to be related to hormonal fluctuations after childbirth. Postpartum depression is not the normal worries (blues) that many new mothers experience. Many caregivers believe that postpartum depression is underdiagnosed and underreported. Postpartum psychosis is a rare condition that usually surfaces within 3 weeks of delivery. Hospitalization of the woman is usually necessary for treatment of this disorder. Bipolar disorder is one of the two categories of postpartum psychosis, characterized by both manic and depressive episodes

One of the first symptoms of puerperal infection to assess for in the postpartum woman is a. fatigue continuing for longer than 1 week. b. pain with voiding. c. profuse vaginal bleeding with ambulation. d. temperature of 38° C (100.4° F) or higher after 24 hours

ANS: D Postpartum or puerperal infection is any clinical infection after childbirth. The definition used in the United States continues to be the presence of a fever of 38° C (100.4° F) or higher on 2 successive days of the first 10 postpartum days, starting 24 hours after birth. Fatigue is a later finding associated with infection. Pain with voiding may indicate a UTI, but it is not typically one of the earlier symptoms of infection. Profuse lochia may be associated with endometritis, but it is not the first symptom associated with infection

Compared to the term infant, the preterm infant has a. few blood vessels visible though the skin. b. more subcutaneous fat. c. well-developed flexor muscles. d. greater surface area in proportion to weight.

ANS: D Preterm infants have greater surface area in proportion to their weight. They often have visible blood vessels because their skin is thin and they have less fat. More fat and welldeveloped flexor muscles are characteristic of a more mature infant

Which is true about newborns classified as small for gestational age (SGA)? a. They weigh less than 2500 g. b. They are born before 38 weeks of gestation. c. Placental malfunction is the only recognized cause of this condition. d. They are below the 10th percentile on gestational growth charts.

ANS: D SGA infants are defined as below the 10th percentile in growth when compared with other infants of the same gestational age. SGA is not defined by weight. Infants born before 38 weeks are defined as preterm. There are many causes of SGA babies

A nurse is caring for an SGA newborn. What nursing action is most important? a. Observe for respiratory distress syndrome. b. Observe for and prevent dehydration. c. Promote bonding. d. Prevent hypoglycemia by early and frequent feedings.

ANS: D The SGA infant has poor glycogen stores and is subject to hypoglycemia. Respiratory distress syndrome is seen in preterm infants. Dehydration is a concern for all infants and is not specific for SGA infants. Promoting bonding is a concern for all infants and is not specific for SGA infants.

What teaching does the nurse provide to help new mothers prevent postpartum depression? a. Stay home and avoid outside activities to ensure adequate rest. b. Be the only caregiver for your baby to facilitate infant attachment. c. Keep feelings of sadness and adjustment to your new role to yourself. d. Realize that this is a common occurrence that affects many women.

ANS: D The new mother should understand that postpartum depression is common. Rest is important, but she does not need to confine herself to the house. Others need to help care for the baby so the mother can rest. Women need to be open and discuss their feelings

Because of the premature infant's decreased immune functioning, what nursing diagnosis should the nurse include in a plan of care for a premature infant? a. Delayed growth and development b. Ineffective thermoregulation c. Ineffective infant feeding pattern d. Risk for infection

ANS: D The nurse needs to know that decreased immune functioning increases the risk for infection. The other diagnoses are appropriate for the premature infant but not related directly to immune function

Which nursing action must be initiated first when evidence of prolapsed cord is found? a. Notify the provider. b. Apply a scalp electrode. c. Prepare the mother for an emergency cesarean delivery. d. Reposition the mother with her hips higher than her head

ANS: D The priority is to relieve pressure on the cord. Changing the maternal position will shift the position of the fetus so that the cord is not compressed. The provider needs to be notified but not until the nurse has taken some corrective action. Trying to relieve pressure on the cord should take priority over increasing fetal monitoring techniques. Emergency cesarean delivery may be necessary if relief of the cord is not accomplished, but attempting to relieve the pressure takes priority. Trying to relieve pressure on the cord should be the first priority.

A pregnant woman at 37 weeks of gestation has had ruptured membranes for 26 hours. A cesarean section is performed for failure to progress. The fetal heart rate before birth is 180 beats/min with limited variability. At birth, the newborn has Apgar scores of 6 and 7 at 1 and 5 minutes and is noted to be pale and tachypneic. Based on the maternal history, the cause of this newborn's distress is most likely a. hypoglycemia. b. phrenic nerve injury. c. respiratory distress syndrome. d. sepsis

ANS: D The prolonged rupture of membranes and the tachypnea (before and after birth) both suggest sepsis. There is no evidence of phrenic nerve damage or respiratory distress syndrome. Early signs of sepsis may be difficult to distinguish from other problems such as hypoglycemia, but the prolonged rupture of membranes puts this baby at high risk of sepsis.

Four hours after delivery of a healthy neonate of an insulin-dependent diabetic woman, the baby appears jittery, irritable, and has a high-pitched cry. Which nursing action has top priority? a. Start an intravenous line with D5W. b. Notify the clinician stat. c. Document the event in the nurses' notes. d. Test for blood glucose level.

ANS: D These are signs of hypoglycemia in the newborn. The nurse should test the infant's blood glucose level and then feed the infant if it is low. It is not common practice to give intravenous glucose to a newborn prior to feeding. Feeding the infant is preferable because the formula or breast milk will last longer. The provider needs to be notified after corrective action has been taken. Documentation should occur but is not the priority.

A macrosomic infant is born after a difficult, forceps-assisted delivery. After stabilization, the infant is weighed, and the birth weight is 4550 g (9 pounds, 6 ounces). What action by the nurse is most appropriate? a. Leave the infant in the room with the mother. b. Take the infant immediately to the nursery. c. Perform a gestational age assessment. d. Monitor blood glucose levels frequently

ANS: D This infant is macrosomic (over 4000 g) and is at high risk for hypoglycemia. Blood glucose levels should be monitored frequently, and the infant should be observed closely for signs of hypoglycemia. The infant can stay with the mother, but this is not the best answer since it does not include the close monitoring needed. Regardless of gestational age, this infant is macrosomic.

Which statement by a postpartum woman indicates that teaching about thrombus formation has been effective? a. "I'll stay in bed for the first 3 days after my baby is born." b. "I'll keep my legs elevated with pillows." c. "I'll sit in my rocking chair most of the time." d. "I'll put my support stockings on every morning before rising."

ANS: D Venous congestion begins as soon as the woman stands up. The stockings should be applied before she rises from the bed in the morning. As soon as possible, the woman should ambulate frequently. The mother should avoid knee pillows because they increase pressure on the popliteal space. Sitting in a chair with legs in a dependent position causes pooling of blood in the lower extremities.

During a neonate resuscitation attempt, the neonatologist has ordered 0.1 mL/kg IV epinephrine (adrenaline) in a 1:10,000 concentration to be given stat. The neonate weighs 3000 grams and is 38 centimeters long. How many millimeters (mL) should the nurse administer? Record your answer using one decimal place

Answer: 0.3 Rationale: Epinephrine should be given if heart rate is 60 after 30 seconds of compressions and ventilation. epinephrine: 1:10,000 concentration 0.1 to 0.3 mL/kg IV 3000 grams = 3 kg 3 kg x 0.1 mL/kg = 0.3 mL

A client experienced prolonged labor with prolonged premature rupture of membranes. The nurse would be alert for which condition in the mother and the newborn? A. infection B. hemorrhage C. trauma D. hypovolemia

Answer: A Rationale: Although hemorrhage, trauma, and hypovolemia may be problems, the prolonged labor with the prolonged premature rupture of membranes places the client at high risk for a postpartum infection. The rupture of membranes removes the barrier of amniotic fluid, so bacteria can ascend.

The pediatrician prescribes morphine sulphate 0.2 mg/kg orally q 4 hour for a neonate suffering from drug withdrawal. The neonate weighs 3,800 grams. How much of drug will the nurse give in 24 hours? Record your answer using two decimal places.

Answer: 4.56 Rationale: 3800 grams = 3.8 kg 3.8 kg/kg x 0.20 mg x 6 doses = 4.56 mg in 24 hours

A pregnant client at 24 weeks' gestation comes to the clinic for an evaluation. The client called the clinic earlier in the day stating that she had not felt the fetus moving since yesterday evening. Further assessment reveals absent fetal heart tones. Intrauterine fetal demise is suspected. The nurse would expect to prepare the client for which testing to confirm the suspicion? A. Ultrasound B. Amniocentesis C. Human chorionic gonadotropin (hCG) level D. Triple marker screening

Answer: A Rationale: A client experiencing an intrauterine fetal demise (IUFD) is likely to seek care when she notices that the fetus is not moving or when she experiences contractions, loss of fluid, or vaginal bleeding. History and physical examination frequently are of limited value in the diagnosis of fetal death, since many times the only history tends to be recent absence of fetal movement and no fetal heart beat heard. An inability to obtain fetal heart sounds on examination suggests fetal demise, but an ultrasound is necessary to confirm the absence of fetal cardiac activity. Once fetal demise is confirmed, induction of labor or expectant management is offered to the client. An amniocentesis, hCG level, or triple marker screening would not be used to confirm IUFD.

A pregnant woman is receiving misoprostol to ripen her cervix and induce labor. The nurse assesses the woman closely for which effect? A. uterine hyperstimulation B. headache C. blurred vision D. hypotension

Answer: A Rationale: A major adverse effect of the obstetric use of misoprostol is hyperstimulation of the uterus, which may progress to uterine tetany with marked impairment of uteroplacental blood flow, uterine rupture (requiring surgical repair, hysterectomy, and/or salpingo-oophorectomy), or amniotic fluid embolism. Headache, blurred vision, and hypotension are associated with magnesium sulfate

A nurse is teaching a woman about measures to prevent preterm labor in future pregnancies because the woman just experienced preterm labor with her most recent pregnancy. The nurse determines that the teaching was successful based on which statement by the woman? A. "I'll make sure to limit the amount of long distance traveling I do." B. "Stress isn't a problem that is related to preterm labor." C. "Separating pregnancies by about a year should be helpful." D. "I'll need extra iron in my diet so I have extra for the baby."

Answer: A Rationale: Appropriate measures to reduce the risk for preterm labor include: avoiding travel for long distances in cars, trains, planes or buses; achieving adequate iron store through balanced nutrition (excess iron is not necessary); waiting for at least 18 months between pregnancies, and using stress management techniques for stress

A client has given birth to a full-term infant weighing 10 pounds 5 ounces (4678 grams). What priority assessment should be completed by the nurse? A. Blood glucose B. Temperature control C. Feeding difficulty D. Perfusion

Answer: A Rationale: Hypoglycemia is a common concern with a large-for-gestational age (LGA) infant. This infant will deplete the glucose stores very rapidly. Therefore, it is important to assess the glucose level within 30 minutes of birth and to repeat every hour until stable. Hypoglycemia is defined as a gluose level less than 35 to 45 mg/dl (1.94 to 2.50 mmol/l) in the first 4 hours of life, and intervention should occur when the glucose is less than 40 mg/dl (2.22 mmol/l). Intervention should also occur if the blood glucose is less than 45 mg/dl (2.50 mmol/l) at 4 and 24 hours of life respectively. Generally the nurse assesses symptoms of jitteriness, irritability and tachypnea first. These symptoms can progress to temperature instability, lethargy, bradycardia, hyponia and seizures

A newborn with severe meconium aspiration syndrome (MAS) is not responding to conventional treatment. Which measure would the nurse anticipate as possibly necessary for this newborn? A. extracorporeal membrane oxygenation (ECMO) B. respiratory support with a ventilator C. insertion of a laryngoscope for deep suctioning D. replacement of an endotracheal tube via X-ray

Answer: A Rationale: If conventional measures are ineffective, then the nurse would need to prepare the newborn for ECMO. Hyperoxygenation, ventilatory support, and direct tracheal suctioning are typically used initially to promote tissue perfusion. However, if these are ineffective, ECMO would be the next step.

A newborn infant has been diagnosed with persistent pulmonary hypertension of the newborn (PPHN). In providing care for this newborn what intervention should be the nurse's priority? A. Measure blood pressure B. Obtain arterial blood gases C. Monitor oxygen saturation D. Suction the newborn

Answer: A Rationale: PPHN occurs when there is persistent fetal circulation after birth. The pulmonary pressures do not decrease at birth when the newborn begins breathing causing hypoxemia, acidosis and vasoconstriction of the pulmonary artery. This newborn requires much care and possibly extracorporeal membrane oxygenation (ECMO). The nurse should monitor the newborn's blood pressure regularly, because hypotension can occur from ensuing heart failure and the persistent hypoxemia. Vasopressors may be needed for this newborn. The newborn should not be suctioned. Doing so causes more stimulation and worsens respiratory issues. Arterial blood gases will be obtained regularly, but they are not a priority nursing intervention. Oxygen saturation should always be monitored in a newborn with respiratory distress

. A nurse is making a home visit to a postpartum client. Which finding would lead the nurse to suspect that a woman is experiencing postpartum psychosis? A. delirium B. feelings of guilt C. sadness D. insomnia

Answer: A Rationale: Postpartum psychosis is at the severe end of the continuum of postpartum emotional disorders. It is manifested by depression that escalates to delirium, hallucinations, anger toward self and infant, bizarre behavior, mania, and thoughts of hurting herself and the infant. Feelings of guilt, sadness, and insomnia are associated with postpartum depression.

On a follow-up visit to the clinic, a nurse suspects that a postpartum client is experiencing postpartum psychosis. Which finding would most likely lead the nurse to suspect this condition? A. delusional beliefs B. feelings of anxiety C. sadness D. insomnia

Answer: A Rationale: Postpartum psychosis is at the severe end of the continuum of postpartum emotional disorders. It is manifested by depression that escalates to delirium, hallucinations, delusional beliefs, anger toward self and infant, bizarre behavior, mania, and thoughts of hurting herself and the infant. Feelings of anxiety, sadness, and insomnia are associated with postpartum depression

A multipara client develops thrombophlebitis after birth. Which assessment findings would lead the nurse to intervene immediately? A. dyspnea, diaphoresis, hypotension, and chest pain B. dyspnea, bradycardia, hypertension, and confusion C. weakness, anorexia, change in level of consciousness, and coma D. pallor, tachycardia, seizures, and jaundice

Answer: A Rationale: Sudden unexplained shortness of breath and reports of chest pain along with diaphoresis and hypotension suggest pulmonary embolism, which requires immediate action. Other signs and symptoms include tachycardia, apprehension, hemoptysis, syncope, and sudden change in the woman's mental status secondary to hypoxemia. Anorexia, seizures, and jaundice are unrelated to a pulmonary embolism.

The nurse is providing care to several pregnant women who may be scheduled for labor induction. The nurse identifies the woman with which Bishop score as having the best chance for a successful induction and vaginal birth? A. 11 B. 7 C. 5 D. 3

Answer: A Rationale: The Bishop score helps identify women who would be most likely to achieve a successful induction. The duration of labor is inversely correlated with the Bishop score: a score over 8 indicates a successful vaginal birth. Therefore the woman with a Bishop score of 11 would have the greatest chance for success. Bishop scores of less than 6 usually indicate that a cervical ripening method should be used prior to induction.

While reviewing a newborn's medical record, the nurse notes that the chest X-ray shows a ground glass pattern. The nurse interprets this as indicative of: A. respiratory distress syndrome. B. transient tachypnea of the newborn. C. asphyxia. D. persistent pulmonary hypertension.

Answer: A Rationale: The chest X-ray of a newborn with RDS reveals a reticular (ground glass) pattern. For TTN, the chest X-ray shows lung overaeration and prominent perihilar interstitial markings and streakings. A chest X-ray for asphyxia would reveal possible structural abnormalities that might interfere with respiration, but the results are highly variable. An echocardiogram would be done to evaluate persistent pulmonary hypertension.

A nurse is developing the plan of care for a small-for-gestational-age newborn. Which action would the nurse determine as a priority? A. Preventing hypoglycemia with early feedings B. Observing for newborn reflexes C. Promoting bonding between the parents and the newborn D. Monitoring vital signs every 2 hours

Answer: A Rationale: The nurse must consider the implications of a small-for-gestational-age newborn. With the loss of the placenta at birth, the newborn must now assume control of glucose homeostasis. This is achieved by early oral intermittent feedings. Observing for newborn reflexes, promoting bonding, and monitoring vital signs, although important, are not the priority for this newborn.

A neonate is admitted to the newborn observation nursery with the possible diagnosis of polycythemia. The nurse would be observing for which findings? Select all that apply. A. ruddy skin color B. respiratory distress C. cyanosis D. pink gums and tongue E. jitteriness

Answer: A, B, C, E Rationale: Observe for clinical signs of polycythemia (respiratory distress, cyanosis, jitteriness, jaundice, ruddy skin color, and lethargy) and monitor blood results

When assessing the postpartum woman, the nurse uses indicators other than pulse rate and blood pressure for postpartum hemorrhage because: A. these measurements may not change until after the blood loss is large. B. the body's compensatory mechanisms activate and prevent any changes. C. they relate more to change in condition than to the amount of blood lost. D. maternal anxiety adversely affects these vital signs.

Answer: A Rationale: The typical signs of hemorrhage do not appear in the postpartum woman until as much as 1,800 to 2,100 ml of blood has been lost. In addition, accurate determination of actual blood loss is difficult because of blood pooling inside the uterus and on perineal pads, mattresses, and the floor.

Assessment of a postpartum client reveals a firm uterus with bright-red bleeding and a localized bluish bulging area just under the skin at the perineum. The woman also reports significant pelvic pain and is experiencing problems with voiding. The nurse suspects which condition? A. hematoma B. laceration C. bladder distention D. uterine atony

Answer: A Rationale: The woman most likely has a hematoma based on the findings: firm uterus with bright-red bleeding; localized bluish bulging area just under the skin surface in the perineal area; severe perineal or pelvic pain; and difficulty voiding. A laceration would involve a firm uterus with a steady stream or trickle of unclotted bright-red blood in the perineum. Bladder distention would be palpable along with a soft, boggy uterus that deviates from the midline. Uterine atony would be noted by a uncontracted uterus.

A pregnant woman gives birth to a small for gestational age neonate who is admitted to the neonatal intensive care unit with seizure activity. The neonate appears to have abnormally small eyes and a thin upper lip. The infant is noted to be microcephalic. Based on these findings, which substance would the nurse suspect the women of using during pregnancy? A. alcohol B. cocaine C. heroin D. methamphetamine

Answer: A Rationale: This child's features match those of fetal alcohol syndrome, including microcephaly, small palpebral (eyelid) fissures, abnormally small eyes, and fetal growth restriction.

A newborn is exhibiting symptoms of withdrawal and toxicology test have been prescribed. Which type of specimen should the nurse collect to obtain the most accurate results? A. Meconium B. Blood C. Urine D. Sputum

Answer: A Rationale: Toxicology screening of a newborn can include testing from blood, urine and meconium. These tests identify which drugs the newborn has been exposed to in utero. A meconium sample can detect which drugs the mother has been using from the second trimester of pregnancy until birth. It is the preferred method of testing. A urine screen identifies only the drugs the mother has used recently. The nurse should be careful not to mix the meconium sample with urine as it alters the results of the test. Blood samples can be taken and they will yield results of current drugs in the newborn's system, but they are invasive and noninvasive testing will provide the same results. If possible, testing on the mother is preferred. This testing provides quick results of what drugs the mother has been exposing the fetus to in utero. This will help in planning and providing care for the drug-exposed newborn. Sputum is not used for toxicology studies

A preterm newborn has received large concentrations of oxygen therapy during a 3-month stay in the NICU. As the newborn is prepared to be discharged home, the nurse anticipates a referral for which specialist? A. ophthalmologist B. nephrologist C. cardiologist D. neurologist

Answer: A Rationale: Use of large concentrations of oxygen and sustained oxygen saturations higher than 95% while on supplemental oxygen have been associated with the development of retinopathy of prematurity (ROP) and further respiratory complications in the preterm newborn (Martin & Deakins, 2020). For these reasons, oxygen should be used judiciously to prevent the development of further complications. A guiding principle for oxygen therapy is it should be targeted to levels appropriate to the condition, gestational age, and postnatal age of the newborn. As a result, an ophthalmology consult for follow-up after discharge is essential for preterm infants who have received extensive oxygen. Although referrals to other specialists may be warranted depending on the newborn's status, there is no information to suggest that any would be needed

Which action would be most appropriate for the nurse to take when a newborn has an unexpected anomaly at birth? A. Show the newborn to the parents as soon as possible while explaining the defect. B. Remove the newborn from the birthing area immediately. C. Inform the parents that there is nothing wrong at the moment. D. Tell the parents that the newborn must go to the nursery immediately.

Answer: A Rationale: When an anomaly is identified at or after birth, parents need to be informed promptly and given a realistic appraisal of the severity of the condition, the prognosis, and treatment options so that they can participate in all decisions concerning their child. Removing the newborn from the area or telling them that the newborn needs to go to the nursery immediately is inappropriate and would only add to the parents' anxieties and fears. Telling them that nothing is wrong is inappropriate because it violates their right to know

A pregnant client has received dinoprostone. Following administration of this medication, the nurse assesses the client and determines that the client is experiencing an adverse effect of the medication based on which client report? Select all that apply. A. headache B. nausea C. diarrhea D. tachycardia E. hypotension

Answer: A, B, C Rationale: Adverse effects associated with dinoprostone include headache, nauseas and vomiting, and diarrhea. Tachycardia and hypotension are not associated with this drug.

. A postpartum woman is diagnosed with endometritis. The nurse interprets this as an infection involving which area? Select all that apply. A. endometrium B. decidua C. myometrium D. broad ligament E. ovaries F. fallopian tubes

Answer: A, B, C Rationale: Endometritis is an infectious condition that involves the endometrium, decidua, and adjacent myometrium of the uterus. Extension of endometritis can result in parametritis, which involves the broad ligament and possibly the ovaries and fallopian tubes, or septic pelvic thrombophlebitis

A premature, 36-week-gestation neonate is admitted to the observational nursery and placed under bili-lights with evidence of hyperbilirubinemia. Which assessment findings would the neonate demonstrate? Select all that apply. A. increased serum bilirubin levels B. clay-colored stools C. tea-colored urine D. cyanosis E. Mongolian spots

Answer: A, B, C Rationale: Hyperbilirubinemia is indicated when the newborn presents with elevated serum bilirubin levels, tea-colored urine, and clay-colored stools. Cyanosis would not be seen in infants in this scenario. Mongolian spots are not associated with newborn jaundice

A preterm newborn is admitted to the neonatal intensive care with the diagnosis of an omphalocele. What nursing actions would the nurse perform? Select all that apply. A. The abdominal contents are protected. B. Fluid loss of the neonate will be minimized. C. Perfusion to the exposed abdominal contents will be maintained. D. Neonatal weight loss will be prevented. E. Assessment of hyperbilirubinemia will be monitored.

Answer: A, B, C Rationale: Nursing management of newborns with omphalocele or gastroschisis focuses on preventing hypothermia, maintaining perfusion to the eviscerated abdominal contents by minimizing fluid loss, and protecting the exposed abdominal contents from trauma and infection. Weight loss at this point is not a priority, and neither is assessing bilirubin.

A nurse is taking a history on a woman who is at 20 weeks' gestation. The woman reports that she feels some heaviness in her thighs since yesterday. The nurse suspects that the woman may be experiencing preterm labor based on which additional assessment findings? Select all that apply. A. dull low backache B. viscous vaginal discharge C. dysuria D. constipation E. occasional cramping

Answer: A, B, C Rationale: Symptoms of preterm labor are often subtle and may include change or increase in vaginal discharge with mucus, water, or blood in it; pelvic pressure; low, dull backache; nausea, vomiting or diarrhea, and heaviness or aching in the thighs. Constipation is not known to be a sign of preterm labor. Preterm labor is assessed when there are more than six contractions per hour. Occasional asymptomatic cramping can be normal.

A 33 weeks' gestation neonate is being assessed for necrotizing enterocolitis (NEC). Which nursing actions would the nurse implement? Select all that apply. A. Perform hemoccult tests on stools. B. Monitor abdominal girth. C. Measure gastric residual before feeds. D. Assess bowel sounds before each feed. E. Assess urine output

Answer: A, B, C, D Rationale: Always keep the possibility of NEC in mind when dealing with preterm newborns, especially when enteral feedings are being administered. Note feeding intolerance, diarrhea, bilestained emesis, or grossly bloody stools. Perform hemoccult tests on the bloody stool. Assess the neonate's abdomen for distention, tenderness, and visible loops of bowel. Measure the abdominal circumference, noting an increase. Listen to bowel sounds before each feeding. Determine residual gastric volume prior to feeding; when it is elevated, be suspicious for NEC. Assessing urine output is not essential.

A macrosomic infant in the newborn nursery is being observed for a possible fractured clavicle. For which would the nurse assess? Select all that apply. A. facial grimacing with movement B. bruising over area C. asymmetrical movement D. edema present E. positive Babinski reflex

Answer: A, B, C, D Rationale: Birth trauma for LGA newborns would be demonstrated by an obvious deformity, with bruising at the site and edema noted. There would be asymmetrical movement when the newborn moves the limb. Babinski reflex is a neurological test and would be normal to be positive.

The nurse in the neonatal intensive care unit is caring for a neonate she suspects is at risk for an intraventricular hemorrhage (IVH). Which nursing actions would be priorities? Select all that apply. A. Maintain fetal flexed position. B. Administer fluids slowly. C. Assess for bulging fontanel. D. Measure head circumference daily. E. Assess Moro reflex. F. Measure intake and output.

Answer: A, B, C, D Rationale: Care of the newborn with IVH is primarily supportive. Correct anemia, acidosis, and hypotension with fluids and medications. Administer fluids slowly to prevent fluctuations in blood pressure. Avoid rapid volume expansion to minimize changes in cerebral blood flow. Keep the newborn in a flexed, contained position with the head elevated to prevent or minimize fluctuations in intracranial pressure. Continuously monitor the newborn for signs of hemorrhage, such as changes in the level of consciousness, bulging fontanel, seizures, apnea, and reduced activity level. Also, measuring head circumference daily to assess for expansion in size is essential in identifying complications early. Moro reflex and intake and output are routine and not associated with IVH

At a preconception counseling class, a client expresses concern and wonders how Healthy People 2030 will improve maternal-infant outcomes. Which response(s) by the nurse is appropriate? Select all that apply. A. Healthy People 2030 will reduce the rate of fetal and infant deaths. B. Healthy People 2030 will decrease the number of all infant deaths (within 1 year). C. Healthy People 2030 will decrease the number of neonatal deaths (within the first year). D. Healthy People 2030 will foster early and consistent prenatal care.

Answer: A, B, C, D Rationale: One of the leading health indicators as identified by Healthy People 2030 refers to decreasing the number of infant deaths. Acquired and congenital conditions account for a significant percentage of infant deaths. Specific objectives include reducing the rate of fetal deaths at 20 or more weeks of gestation though the nursing action of fostering early and consistent prenatal care; reducing the rate of all infant deaths (within 1 year) through the nursing actions of including education to place infants on their backs for naps and sleep to prevent sudden infant death syndrome (SIDS), avoiding exposing newborns to cigarette smoke, and ensuring that infants with birth defects receive health care needed in order to thrive; and reducing the occurrence of fetal alcohol syndrome (FAS) through the nursing actions or counseling girls and women to avoid alcohol use during pregnancy, and participating in programs for at-risk groups, including adolescents, about the effects of substance use, especially alcohol, during pregnancy.

A term neonate has been admitted to the observational newborn nursery with the diagnosis of being small for gestational age. Which factors would predispose the neonate to this diagnosis? Select all that apply. A. The mother had chronic placental abruption. B. At birth the placenta was noted to be decreased in weight. C. On assessment the placenta had areas of infarction. D. At birth the placenta was a shiny Schultz presentation. E. Placental talipes was present at birth

Answer: A, B, C, D Rationale: Placental factors that can contribute to a small for gestational age infant include chronic placental abruption, infarction on surface of placenta, and a decreased placental weight. A shiny Schultz placenta is a normal description because the fetal side of the placenta comes out first, which is shiny. Placenta talipes does not exist.

A nurse is massaging a postpartum client's fundus and places the nondominant hand on the area above the symphysis pubis based on the understanding that this action: A. determines that the procedure is effective. B. helps support the lower uterine segment. C. aids in expressing accumulated clots. D. prevents uterine muscle fatigue

Answer: B Rationale: The nurse places the nondominant hand on the area above the symphysis pubis to help support the lower uterine segment. The hand, usually the dominant hand that is placed on the fundus, helps to determine uterine firmness (and thus the effectiveness of the massage). Applying gentle downward pressure on the fundus helps to express clots. Overmassaging the uterus leads to muscle fatigue

A 20-hour-old neonate is suspected of having polycythemia. Which nursing intervention(s) will the nurse utilize to provide care for this neonate? Select all that apply. A. Obtain hemoglobin and hematocrit laboratory tests B. Provide early feedings to prevent hypoglycemia C. Maintain oxygen saturation parameters D. Monitor urinary output E. Insert a peripheral IV

Answer: A, B, C, D Rationale: Polycythemia in a neonate is defined as a hematocrit above 65% (0.65) and a hemoglobin level above 20 g/dl (200 g/l). The hematocrit and hemoglobin peak between 6 and 12 hours of life and then start to decrease. If these values do not decrease as expected, then hypoperfusion will occur and polycythemia will develop. In the beginning, the nurse may assess feeding difficulties, hypoglycemia, jitteriness and respiratory distress. As the condition worsens, a ruddy skin color could be seen, cyanosis could develop, the neonate could become lethargic and seizures could develop. Nursing care for this neonate requires obtaining hematocrit and hemoglobin laboratory tests at 2 hours, 12 hours and 24 hours. Feeding should be started to provide fluid, nutrition and prevent hypoglycemia. The oxygen saturation should be monitored. If the levels are below the established parameters from the health care provider, oxygen therapy will be needed. The urine output should be monitored continuously because polycythemia can

A neonate is born at 42 weeks' gestation weighing 4.4 kg (9 lb, 7 oz) with satisfactory Apgar scores. Two hours later birth the neonate's blood sugar indicates hypoglycemia. Which symptoms would the baby demonstrate? Select all that apply. A. poor sucking B. respiratory distress C. weak cry D. jitteriness E. blood glucose >40 mg/dl

Answer: A, B, C, D Rationale: Some of the common problems associated with newborns experiencing a variation in gestational age, such as a postterm newborn, are respiratory distress, jitteriness, feeble sucking, weak cry, and a blood glucose of 40 mg/dl.

A client expresses concerns that her grandmothers had complicated pregnancies. What principle(s) should the nurse discuss to allay the fears of the client? Select all that apply. A. "We work to ensure that birth of high-risk infants happens in settings where we are able to care for them." B. "We will work with you to identify prenatal risk factors early and take actions to reduce their impact." C. "We support those at risk of having a preterm births with the goal of delaying early births." D. "We work to ensure care for mothers and infants to reduce infant illnesses, disabilities, and death." E. "We allow families to grieve the loss of a newborn, should it occur."

Answer: A, B, C, D Rationale: The nurse will attempt to allay the client's fears by discussing the actions the facility enacts to promote a healthy brith and infant. This includes ensuring the birth of high-risk infants takes place in settings that have the technological capacity to care for them, identifying risk factors early and taking action to reduce their impact, working to delay the birth of those pregancies identified at risk of preterm birht, and promoting an overall reduction in infant illness, disability, and death to proper care of the mother and infant. Although allowing a family to greive in instances of infant death, discussing this factor with the client is likely to create more fear

A nurse is reading a journal article about cesarean births and the indications for them. Place the indications for cesarean birth below in the proper sequence from most frequent to least frequent. All options must be used. A. Labor dystocia B. Abnormal fetal heart rate tracing C. Fetal malpresentation D. Multiple gestation E. Suspected macrosomia

Answer: A, B, C, D, E Rationale: The most common indications for primary cesarean births include, in order of frequency: labor dystocia as the labor does not progress, abnormal fetal heart rate tracing indicating fetal distress, fetal malpresentation making a difficult progression of labor, multiple gestation , and suspected macrosomia.

. A pregnant client at 30 weeks' gestation calls the clinic because she thinks that she may be in labor. To determine if the client is experiencing labor, which question(s) would be appropriate for the nurse to ask? Select all that apply. A. "Are you feeling any pressure or heaviness in your pelvis?" B. "Are you having contractions that come and go, off and on?" C. "Have you noticed any fluid leaking from your vagina?" D. "Are you having problems with heartburn?" E. "Have you been having any nausea or vomiting?"

Answer: A, B, C, E Rationale: Frequently, women are unaware that uterine contractions, effacement, and dilation are occurring, thus making early intervention ineffective in arresting preterm labor and preventing the birth of a premature newborn. The nurse should ask the client about any signs/symptoms, being alert for subtle symptoms of preterm labor, which may include: a change or increase in vaginal discharge with mucous, water, or blood in it; pelvic pressure (pushing-down sensation); low dull backache; menstrual-like cramps; urinary tract infection symptoms; feeling of pelvic pressure or fullness; gastrointestinal upset like nausea, vomiting, and diarrhea; general sense of discomfort or unease; heaviness or aching in the thighs; uterine contractions with or without pain; more than six contractions per hour; intestinal cramping with or without diarrhea. Contractions also must be persistent, such that four contractions occur every 20 minutes or eight contractions occur in 1 hour. A report of heartburn is unrelated to preterm labor.

A nurse is conducting an in-service program for a group of labor and birth unit nurses about cesarean birth. The group demonstrates understanding of the information when they identify which conditions as appropriate indications? Select all that apply. A. active genital herpes infection B. placenta previa C. previous cesarean birth D. prolonged labor E. fetal distress

Answer: A, B, C, E Rationale: The leading indications for cesarean birth are previous cesarean birth, breech presentation, dystocia, and fetal distress. Examples of specific indications include active genital herpes, fetal macrosomia, fetopelvic disproportion, prolapsed umbilical cord, placental abnormality (placenta previa or placental abruption), previous classic uterine incision or scar, gestational hypertension, diabetes, positive human immunodeficiency virus (HIV) status, and dystocia. Fetal indications include malpresentation (nonvertex presentation), congenital anomalies (fetal neural tube defects, hydrocephalus, abdominal wall defects), and fetal distress.

A 42-year-old woman is 26 weeks' pregnant. She lives at a shelter for female victims of intimate partner violence. Her blood pressure is 170/90 mm Hg, the fetal heart rate is 140 bpm, TORCH studies are positive, and she is bleeding vaginally. What findings put her at risk of giving birth to a small-for-gestational-age (SGA) infant? Select all that apply. A. the age of the client B. living in a shelter for victims of intimate partner violence C. vaginal bleeding D. fetal heart rate E. blood pressure F. positive test for TORCH

Answer: A, B, C, E, F Rationale: Some factors contributing to the birth of SGA newborns include maternal age of 20 or 35 years old, low socioeconomic status, and preeclampsia with increased blood pressure. The vaginal bleeding indicates placental problems, and she tests positive for sexually transmitted diseases by TORCH group infections.

A small-for-gestational age infant is admitted to the observational care unit with the nursing diagnosis of ineffective thermoregulation related to lack of fat stores as evidenced by persistent low temperatures. Which are appropriate nursing interventions? Select all that apply. A. Assess the axillary temperature every hour. B. Review maternal history. C. Assess environment for sources of heat loss. D. Bathe the neonate with warmer water. E. Minimize kangaroo care. F. Encourage skin-to-skin contact.

Answer: A, B, C, F Rationale: Proper care to promote thermoregulation include assessing the axillary temperature every hour, reviewing the maternal history to identify risk factors contributing to problem, assessing the environment for sources of heat loss, avoiding bathing and exposing newborn to prevent cold stress, and encouraging kangaroo care (mother or father holds preterm infant underneath clothing skin-to-skin and upright between breasts) to provide warmth.

A 30 weeks' gestation neonate born with low Apgar scores is in the neonatal intensive care unit with respiratory distress syndrome and underwent an exchange transfusion for anemia. Which factors place the neonate at risk for necrotizing enterocolitis? Select all that apply A. preterm birth B. respiratory distress syndrome C. low Apgar scores D. hyperthermia E. hyperglycemia F. exchange transfusion

Answer: A, B, C, F Rationale: The predisposing factors for the development of necrotizing enterocolitis include preterm labor, respiratory distress syndrome, exchange transfusion, and low birth weight. Low Apgar scores, hypothermia, and hypoglycemia are also risk factors

A 2-hour-old neonate born via caesarean birth has begun having a respiratory rate of 110 breaths/min and is in respiratory distress. What intervention(s) is a priority for the nurse to include in this neonates's care? Select all that apply. A. Keep the head in a "sniff" position B. Administer oxygen C. Insert an orogastric tube D. Ensure thermoregulation E. Obtain an arterial blood gas

Answer: A, B, D Rationale: This neonate is experiencing manifestations of transient tachypnea of the newborn (TTN). It occurs from delayed clearing of the lungs from fluid, and can be seen in neonates born via cesarean birth, because they have not had the experience of the compression on the thorax during vaginal delivery. This starts within the first 6 hours of life and can last up to 72 hours. The priority interventions for this neonate are oxygen, thermoregulation and minimal stimulation. Keeping the head in a neutral or "sniff " position allows for optimal airway. If the neonate becomes cold, then respiratory distress and or sepsis can develop. Minimal stimulation conserves the neonate's respiratory and heat requirements. The neonate may need placement of a peripheral IV for hydration and/or a feeding tube for formula or breast milk. The neonate should not be nipple fed until the respirations are under 60 breaths/min. A chest x-ray and an arterial blood gas may be needed also, but they would only be necessary if the neonate is in severe distress. The arterial blood gas results would show mild hypoxemia, a midly elevated CO2 level, and a normal pH.

A nurse is conducting a review course on tocolytic therapy for perinatal nurses. After teaching the group, the nurse determines that the teaching was successful when they identify which drugs as being used for tocolysis? Select all that apply. A. nifedipine B. magnesium sulfate C. dinoprostone D. misoprostol E. indomethacin

Answer: A, B, E Rationale: Medications most commonly used for tocolysis include magnesium sulfate (which reduces the muscle's ability to contract), indomethacin (a prostaglandin synthetase inhibitor), and nifedipine (a calcium channel blocker). These drugs are used "off label": this means they are effective for this purpose but have not been officially tested and developed for this purpose by the FDA. Dinoprostone and misoprostol are used to ripen the cervix.

A nurse is providing a refresher class for a group of postpartum nurses. The nurse reviews the risk factors associated with postpartum hemorrhage. The group demonstrates understanding of the information when they identify which risk factors associated with uterine tone? Select all that apply. A. rapid labor B. retained blood clots C. hydramnios D. operative birth E. fetal malpostion

Answer: A, C Rationale: Risk factors associated with uterine tone include hydramnios, rapid or prolonged labor, oxytocin use, maternal fever, or prolonged rupture of membranes. Retained blood clots are a risk factor associated with tissue retained in the uterus. Fetal malposition and operative birth are risk factors associated with trauma of the genital tract.

The nurse is developing a discharge teaching plan for a postpartum woman who has developed a postpartum infection. Which measures would the nurse most likely include in this teaching plan? Select all that apply. A. taking the prescribed antibiotic until it is finished B. checking temperature once a week C. washing hands before and after perineal care D. handling perineal pads by the edges E. directing peribottle to flow from back to front

Answer: A, C, D Rationale: Teaching should address taking the prescribed antibiotic until finished to ensure complete eradication of the infection; checking temperature daily and notifying the practitioner if it is above 100.4° F (38° C); washing hands thoroughly before and after eating, using the bathroom, touching the perineal area, or providing newborn care; handling perineal pads by the edges and avoiding touching the inner aspect of the pad that is against the body; and directing peribottle so that it flows from front to back.

The nurse notifies the obstetrical team immediately because the nurse suspects that the pregnant woman may be exhibiting signs and symptoms of amniotic fluid embolism. When reporting this suspicion, which finding(s) would the nurse include in the report? Select all that apply. A. significant difficulty breathing B. hypertension C. tachycardia D. pulmonary edema E. bleeding with bruising

Answer: A, C, D, E Rationale: Anaphylactoid syndrome of pregnancy (ASP), also known as amniotic fluid embolism, is an unforeseeable, life-threatening complication of childbirth. The etiology of ASP remains an enigmatic, devastating obstetric condition associated with significant maternal and newborn morbidity and mortality. It is a rare and often fatal event characterized by the sudden onset of hypotension, cardiopulmonary collapse, hypoxia, and coagulopathy. ASP should be suspected in any pregnant women with an acute onset of dyspnea, hypotension, and DIC. By knowing how to intervene, the nurse can promote a better chance of survival for both the mother and her newborn

A 32-year-old black woman in her second trimester has come to the clinic for an evaluation. While interviewing the client, she reports a history of fibroids and urinary tract infection. The client states, "I know smoking is bad and I have tried to stop, but it is impossible. I have cut down quite a bit though, and I do not drink alcohol." Complete blood count results reveal a low red blood cell count, low hemoglobin, and low hematocrit. When planning this client's care, which factor(s) would the nurse identify as increasing the client's risk for preterm labor? Select all that apply. A. African heritage B. Maternal age C. History of fibroids D. Cigarette smoking E. History of urinary tract infections F. Complete blood count results

Answer: A, C, D, E, F Rationale: For this client, risk factors associated with preterm labor and birth would include African heritage, cigarette smoking, uterine abnormalities, such as fibroids, urinary tract infection, and possible anemia based on her complete blood count results. Maternal age extremes (younger than 16 years and older than 35 years) are also a risk factor but do not apply to this client.

A 22-year-old woman experiencing homelessness arrives at a walk-in clinic seeking pregnancy confirmation. The nurse notes on assessment her uterus suggests 12 weeks' gestation, a blood pressure of 110/70 mm Hg, and a BMI of 17.5. The client admits to using cocaine a few times. The client has been pregnant before and indicates she "loses them early." What characteristic(s) place the client in the high-risk pregnancy category? Select all that apply. A. BMI 17.5 B. blood pressure 110/70 mm Hg C. prenatal history D. homelessness E. age F. prenatal care

Answer: A, C, D, F Rationale: The key to identifying a newborn with special needs related to birthweight or gestational age variation is an awareness of the factors that could place a newborn at risk. These factors are similar to those that would suggest a high-risk pregnancy and include maternal nutrition (malnutrition or overweight), substandard living conditions or low socioeconomic status, maternal age of less than 20 or more than 35 years, lack of prenatal care, and history of previous preterm birth.

Assessment of a postpartum woman experiencing postpartum hemorrhage reveals mild shock. Which finding would the nurse expect to assess? Select all that apply. A. diaphoresis B. tachycardia C. oliguria D. cool extremities E. confusion

Answer: A, D Rationale: Signs and symptoms of mild shock include diaphoresis, increased capillary refill, cool extremities, and maternal anxiety. Tachycardia and oliguria suggest moderate shock. Confusion suggests severe shock.

A home health care nurse is assessing a postpartum woman who was discharged 2 days ago. The woman tells the nurse that she has a low-grade fever and feels "lousy." Which finding would lead the nurse to suspect endometritis? Select all that apply. A. lower abdominal tenderness B. urgency C. flank pain D. breast tenderness E. anorexia

Answer: A, E Rationale: Manifestations of endometritis include lower abdominal tenderness or pain on one or both sides, elevated temperature, foul-smelling lochia, anorexia, nausea, fatigue and lethargy, leukocytosis, and elevated sedimentation rate. Urgency and flank pain would suggest a urinary tract infection. Breast tenderness may be related to engorgement or suggest mastitis.

A woman receives magnesium sulfate as treatment for preterm labor. The nurse assess and maintains the infusion at the prescribed rate based on which finding? A. Respiratory rate-16 breaths/minute B. Decreased fetal heart rate variability C. Urine output 22 mL/hour D. Absent deep tendon reflexes

Answer: B Rationale: A respiratory rate of 16 breaths per minute is appropriate and within acceptable parameters to continue the infusion. When administering magnesium sulfate, the nurse would immediately report decreaed fetal heart rate variability, a urine output less than 30 mL/hour, and decreased or absent deep tendon reflexes

A nurse is developing a program to help reduce the risk of late postpartum hemorrhage in clients in the labor and birth unit. Which measure would the nurse emphasize as part of this program? A. administering broad-spectrum antibiotics B. inspecting the placenta after delivery for intactness C. manually removing the placenta at birth D. applying pressure to the umbilical cord to remove the placenta

Answer: B Rationale: After the placenta is expelled, a thorough inspection is necessary to confirm its intactness because tears or fragments left inside may indicate an accessory lobe or placenta accreta. These can lead to profuse hemorrhage because the uterus is unable to contract fully. Administering antibiotics would be appropriate for preventing infection, not postpartum hemorrhage. Manual removal of the placenta or excessive traction on the umbilical cord can lead to uterine inversion, which in turn would result in hemorrhage

A newborn has an Apgar score of 6 at 5 minutes. Which action would be the priority? A. initiating IV fluid therapy B. beginning resuscitative measures C. promoting kangaroo care D. obtaining a blood culture

Answer: B Rationale: An Apgar score below 7 at 1 or 5 minutes indicates the need for resuscitation. Intravenous fluid therapy and blood cultures may be done once resuscitation is started. Kangaroo care would be appropriate once the newborn is stable

A woman with preterm labor is receiving magnesium sulfate. Which finding would require the nurse to intervene immediately? A. respiratory rate of 16 breaths per minute B. 1+ deep tendon reflexes C. urine output of 45 mL/hour D. alert level of consciousness

Answer: B Rationale: Diminished deep tendon reflexes (1+) suggest magnesium toxicity, which requires immediate intervention. Additional signs of magnesium toxicity include a respiratory rate less than 12 breaths/minute, urine output less than 30 mL/hour, and a decreased level of consciousness.

A nurse is providing care to a newborn who is receiving phototherapy. Which action would the nurse most likely include in the plan of care? A. keeping the newborn in the supine position B. covering the newborn's eyes while under the bililights C. ensuring that the newborn is covered or clothed D. reducing the amount of fluid intake to 8 ounces daily

Answer: B Rationale: During phototherapy, the newborn's eyes are covered to protect them from the lights. The newborn is turned every 2 hours to expose all areas of the body to the lights and is kept undressed, except for the diaper area, to provide maximum body exposure to the lights. Fluid intake is increased to allow for added fluid, protein, and calories

A nurse is describing the risks associated with post-term pregnancies as part of an inservice presentation. The nurse determines thatmore teaching is needed when the group identifies which factor as an underlying reason for problems in the fetus? A. aging of the placenta B. increased amniotic fluid volume C. meconium aspiration D. cord compression

Answer: B Rationale: Fetal risks associated with a post-term pregnancy include macrosomia, shoulder dystocia, brachial plexus injuries, low Apgar scores, postmaturity syndrome (loss of subcutaneous fat and muscle and meconium staining), and cephalopelvic disproportion. As the placenta ages, its perfusion decreases and it becomes less efficient at delivering oxygen and nutrients to the fetus. Amniotic fluid volume also begins to decline after 38 weeks' gestation, possibly leading to oligohydramnios, subsequently resulting in fetal hypoxia and an increased risk of cord compression because the cushioning effect offered by adequate fluid is no longer present. Hypoxia and oligohydramnios predispose the fetus to aspiration of meconium, which is released by the fetus in response to a hypoxic insult (Norwitz, 2019). All of these issues can compromise fetal well-being and lead to fetal distress.

The nurse prepares to administer a gavage feeding for a newborn with transient tachypnea based on the understanding that this type of feeding is necessary because: A. lactase enzymatic activity is not adequate. B. oxygen demands need to be reduced. C. renal solute lead must be considered. D. hyperbilirubinemia is likely to develop.

Answer: B Rationale: For the newborn with transient tachypnea, the newborn's respiratory rate is high, increasing the oxygen demand. Thus, measures are initiated to reduce this demand. Gavage feedings are one way to do so. With transient tachypnea, enzyme activity and kidney function are not affected. This condition typically resolves within 72 hours. The risk for hyperbilirubinemia is not increased.

The nurse is providing care to a newborn who was born at 36 weeks' gestation. Based on the nurse's understanding of gestational age, the nurse identifies this newborn as: A. preterm. B. late preterm. C. term. D. postterm.

Answer: B Rationale: Gestational age is typically measured in weeks: a newborn born before completion of 37 weeks is classified as a preterm newborn, and one born after completion of 42 weeks is classified as a postterm newborn. An infant born from the first day of the 38th week through 42 weeks is classified as a term newborn. The late preterm newborn (near term) is one who is born between 34 weeks and 36 weeks, 6 days of gestation.

A nurse is assessing a postpartum client who is at home. Which statement by the client would lead the nurse to suspect that the client may be developing postpartum depression? A. "I just feel so overwhelmed and tired." B. "I'm feeling so guilty and worthless lately." C. "It's strange, one minute I'm happy, the next I'm sad." D. "I keep hearing voices telling me to take my baby to the river."

Answer: B Rationale: Indicators for postpartum depression include feelings related to restlessness, worthlessness, guilt, hopeless, and sadness along with loss of enjoyment, low energy level, and loss of libido. The statements about being overwhelmed and fatigued and changing moods suggest postpartum blues. The statement about hearing voices suggests postpartum psychosis.

A postpartum woman who developed deep vein thrombosis is being discharged on anticoagulant therapy. After teaching the woman about this treatment, the nurse determines that additional teaching is needed when the woman makes which statement? A. "I will use a soft toothbrush to brush my teeth." B. "I can take ibuprofen if I have any pain." C. "I need to avoid drinking any alcohol." D. "I will call my health care provider if my stools are black and tarry."

Answer: B Rationale: Individuals receiving anticoagulant therapy need to avoid use of any over-the-counter products containing aspirin or aspirin-like derivatives such as NSAIDs (ibuprofen) to reduce the risk for bleeding. Using a soft toothbrush and avoiding alcohol are appropriate measures to reduce the risk for bleeding. Black, tarry stools should be reported to the health care provider.

The nurse prepares to assess a newborn who is considered to be large-for-gestational-age (LGA). Which characteristic would the nurse correlate with this gestational age variation? A. strong, brisk motor skills B. difficulty in arousing to a quiet alert state C. birthweight of 7 lb, 14 oz (3,572 g) D. wasted appearance of extremities

Answer: B Rationale: LGA newborns typically are more difficult to arouse to a quiet alert state. They have poor motor skills, have a large body that appears plump and full-sized, and usually weigh more than 8 lb, 13 oz (3,997 g) at term.

A nurse is explaining to a group of nurses new to the labor and birth unit about methods used for cervical ripening. The group demonstrates understanding of the information when they identify which method as a mechanical one? A. herbal agents B. laminaria C. membrane stripping D. amniotomy

Answer: B Rationale: Laminaria is a hygroscopic dilator that is used as a mechanical method for cervical ripening. Herbal agents are a nonpharmacologic method. Membrane stripping and amniotomy are considered surgical methods

A nurse is reviewing the maternal history of a large-for-gestational-age (LGA) newborn. Which factor, if noted in the maternal history, would the nurse identify as possibly contributing to the birth of this newborn? A. substance use disorder B. diabetes C. preeclampsia D. infection

Answer: B Rationale: Maternal factors that increase the chance of having an LGA newborn include maternal diabetes mellitus or glucose intolerance, multiparity, prior history of a macrosomic infant, postdate gestation, maternal obesity, male fetus, and genetics. Substance use disorder is associated with small-for-gestational-age (SGA) newborns and preterm newborns. A maternal history of preeclampsia and infection would be associated with preterm birth.

A postpartum client is prescribed medication therapy as part of the treatment plan for postpartum hemorrhage. Which medication would the nurse expect to administer in this situation? A. Magnesium sulfate B. methylergonovine C. Indomethacin D. nifedipine

Answer: B Rationale: Methylergonovine, along wiht oxytocin and carboprost are drugs used to manage postpartum hemorrhage. Magnesium sulfate, indomethecin, and nifedipine are used to control preterm labor

A newborn is suspected of developing persistent pulmonary hypertension. The nurse would expect to prepare the newborn for which procedure to confirm the suspicion? A. chest X-ray B. blood cultures C. echocardiogram D. stool for occult blood

Answer: C Rationale: An echocardiogram is used to reveal right-to-left shunting of blood to confirm the diagnosis of persistent pulmonary hypertension. Chest X-ray would be most likely used to aid in the diagnosis of RDS or TTN. Blood cultures would be helpful in evaluating for neonatal sepsis. Stool for occult blood may be done to evaluate for NEC

Which information would the nurse include when teaching a new mother about the difference between pathologic and physiologic jaundice? A. Physiologic jaundice results in kernicterus. B. Pathologic jaundice appears within 24 hours after birth. C. Both are treated with exchange transfusions of maternal O- blood. D. Physiologic jaundice requires transfer to the NICU

Answer: B Rationale: Pathologic jaundice appears within 24 hours after birth whereas physiologic jaundice commonly appears around the third or fourth days of life. Kernicterus is more commonly associated with pathologic jaundice. An exchange transfusion is used only if the total serum bilirubin level remains elevated after intensive phototherapy. With this procedure, the newborn's blood is removed and replaced with nonhemolyzed red blood cells from a donor. Physiologic jaundice often is treated at home

After teaching the parents of a newborn with periventricular hemorrhage about the disorder and treatment, which statement by the parents indicates that the teaching was successful? A. "We'll make sure to cover both of his eyes to protect them." B. "Our newborn could develop a learning disability later on." C. "Once the bleeding ceases, there won't be any more worries." D. "We need to get family members to donate blood for transfusion."

Answer: B Rationale: Periventricular hemorrhage has long-term sequelae such as seizures, hydrocephalus, periventricular leukomalacia, cerebral palsy, learning disabilities, vision or hearing deficits, and intellectual disability. Covering the eyes is more appropriate for the newborn receiving phototherapy. The bleeding in the brain can lead to serious long-term effects. Blood transfusions are not used to treat periventricular hemorrhage.

Which assessment finding will alert the nurse to be on the lookout for possible placental abruption during labor? A. macrosomia B. gestational hypertension C. gestational diabetes D. low parity

Answer: B Rationale: Risk factors for placental abruption include preeclampsia, gestational hypertension, seizure activity, uterine rupture, trauma, smoking, cocaine use, coagulation defects, previous history of abruption, intimate partner violence, and placental pathology. Macrosomia, gestational diabetes, and low parity are not considered risk factors.

A neonate born at 40 weeks' gestation, weighing 2300 grams (5 lb, 1 oz) is admitted to the newborn nursery for observation only. What is the nurse's first observation about the infant? A. The neonate is average for its gestational age. B. The neonate is small for its gestational age. C. The neonate is large for its gestational age. D. The neonate is fetal growth restricted.

Answer: B Rationale: Small for gestational age (SGA) describes newborns that typically weigh less than 2,500 g (5 lb, 8 oz) at term due to less growth than expected in utero. A newborn is also classified as SGA if his or her birthweight is at or below the 10th percentile as correlated with the number of weeks of gestation. In some SGA newborns, the rate of growth does not meet the expected growth pattern. These infants are considered to have fetal growth restriction resulting in pathology

A nurse is teaching the mother of a newborn experiencing cocaine withdrawal about caring for the neonate at home. The mother stopped using cocaine near the end of her pregnancy. The nurse determines that additional teaching is needed when the mother identifies which action as appropriate for her newborn? A. wrapping the newborn snugly in a blanket B. waking the newborn every hour C. checking the newborn's fontanels D. offering a pacifier

Answer: B Rationale: Stimuli need to be decreased. Waking the newborn every hour would most likely be too stimulating. Measures such as swaddling the newborn tightly and offering a pacifier help to decrease irritable behaviors. A pacifier also helps to satisfy the newborn's need for nonnutritive sucking. Checking the fontanels provides evidence of hydration.

A postpartum client comes to the clinic for her routine 6-week visit. The nurse assesses the client and suspects that she is experiencing subinvolution based on which finding? A. nonpalpable fundus B. moderate lochia serosa C. bruising on arms and legs D. fever

Answer: B Rationale: Subinvolution is usually identified at the woman's postpartum examination 4 to 6 weeks after birth. The clinical picture includes a postpartum fundal height that is higher than expected, with a boggy uterus; the lochia fails to change colors from red to serosa to alba within a few weeks. Normally, at 4 to 6 weeks, lochia alba or no lochia would be present and the fundus would not be palpable. Thus evidence of lochia serosa suggests subinvolution. Bruising would suggest a coagulopathy. Fever would suggest an infection.

A nurse is assessing a client who gave birth vaginally about 4 hours ago. The client tells the nurse that she changed her perineal pad about an hour ago. On inspection, the nurse notes that the pad is now saturated. The uterus is firm and approximately at the level of the umbilicus. Further inspection of the perineum reveals an area, bluish in color and bulging just under the skin surface. Which action would the nurse do next? A. Apply warm soaks to the area. B. Notify the health care provider. C. Massage the uterine fundus. D. Encourage the client to void

Answer: B Rationale: The client is experiencing postpartum hemorrhage secondary to a perineal hematoma. The nurse needs to notify the health care provider about these findings to prevent further hemorrhage. Applying warm soaks to the area would do nothing to control the bleeding. With a perineal hematoma, the uterus is firm, so massaging the uterus or encouraging the client to void would not be appropriate

A nurse is reviewing a journal article on the causes of postpartum hemorrhage. Which condition would the nurse most likely find as the most common cause? A. labor augmentation B. uterine atony C. cervical or vaginal lacerations D. uterine inversion

Answer: B Rationale: The most common cause of postpartum hemorrhage is uterine atony, failure of the uterus to contract and retract after birth. The uterus must remain contracted after birth to control bleeding from the placental site. Labor augmentation is a risk factor for postpartum hemorrhage. Lacerations of the birth canal and uterine inversion may cause postpartum hemorrhage, but these are not the most common cause.

. A group of nurses are reviewing information about mastitis and its causes in an effort to develop a teaching program on prevention for postpartum women. The nurses demonstrate understanding of the information when they focus the teaching on ways to minimize risk of exposure to which organism? A. E. coli B. S. aureus C. Proteus D. Klebsiella

Answer: B Rationale: The most common infectious organism that causes mastitis is S. aureus, which comes from the breast-feeding infant's mouth or throat. E. coli is another, less common cause. E. coli, Proteus, and Klebsiella are common causes of urinary tract infections

A late preterm newborn is being prepared for discharge to home after being in the neonatal intensive care unit for 4 days. The nurse instructs the parents about the care of their newborn and emphasizes warning signs that should be reported to the pediatrician immediately. The nurse determines that additional teaching is needed based on which parental statement? A. "We will call 911 if we start to see that our newborn's lips or skin are looking bluish." B. "If our newborn's skin turns yellow, it is from the treatments and our newborn is okay." C. "If our newborn does not have a wet diaper in 12 hours, we will call our pediatrician." D. "We will let the pediatrician know if our newborn's temperature goes above 100.4°F (38°C)."

Answer: B Rationale: The parents of a preterm newborn need teaching about when to notify their pediatrician or nurse practitioner. These include: displaying a yellow color to the skin (jaundice); having difficulty breathing or turning blue (call for emergency services in this case); having a temperature below 97°F (36.1°C) or above 100.4°F (38°C); and failing to void for 12 hours.

When describing the stages of labor to a pregnant woman, which of the following would the nurse identify as the major change occurring during the first stage? A. Regular contractions B. Cervical dilation C. Fetal movement through the birth canal D. Placental separation

Answer: B Rationale: The primary change occurring during the first stage of labor is progressive cervical dilation. Contractions occur during the first and second stages of labor. Fetal movement through the birth canal is the major change during the second stage of labor. Placental separation occurs during the third stage of labor.

A primigravida whose labor was initially progressing normally is now experiencing a decrease in the frequency and intensity of her contractions. The nurse would assess the woman for which condition? A. a low-lying placenta B. fetopelvic disproportion C. contraction ring D. uterine bleeding

Answer: B Rationale: The woman is experiencing dystocia most likely due to hypotonic uterine dysfunction and fetopelvic disproportion associated with a large fetus. A low-lying placenta, contraction ring, or uterine bleeding would not be associated with a change in labor pattern

After teaching a woman with a postpartum infection about care after discharge, which client statement indicates the need for additional teaching? A. "I need to call my doctor if my temperature goes above 100.4° F (38° C)." B. "When I put on a new pad, I'll start at the back and go forward." C. "If I have chills or my discharge has a strange odor, I'll call my doctor." D. "I'll point the spray of the peri-bottle so it the water flows front to back."

Answer: B Rationale: The woman needs additional teaching when she states that she should apply the perineal pad starting at the back and going forward. The pad should be applied using a front-toback motion. Notifying the health care provider of a temperature above 100.4° F (38° C), aiming the peri-bottle spray so that the flow goes from front to back, and reporting danger signs such as chills or lochia with a strange odor indicate effective teaching.

A one-day-old neonate born at 32 weeks' gestation is in the neonatal intensive care unit under a radiant overhead warmer. The nurse assesses the morning axilla temperature as 95 degrees F (35 degrees C). What could explain the assessment finding? A. Conduction heat loss is a problem in the baby. B. The supply of brown adipose tissue is not developed. C. Axillary temperatures are not accurate. D. This is a normal temperature.

Answer: B Rationale: Typically newborns use nonshivering thermogenesis for heat production by metabolizing their own brown adipose tissue. However, this preterm newborn has an inadequate supply of brown fat because he or she left the uterus early before the supply was adequate. Conduction heat loss allows an increased transfer of heat from their bodies to the environment, but there is nothing to substantiate conduction heat loss. Axillary temperatures are accurate and the mode of taking temperatures for neonates

After teaching a couple about what to expect with their planned cesarean birth, which statement indicates the need for additional teaching? A. "Holding a pillow against my incision will help me when I cough." B. "I'm going to have to wait a few days before I can start breastfeeding." C. "I guess the nurses will be getting me up and out of bed rather quickly." D. "I'll probably have a tube in my bladder for about 24 hours or so."

Answer: B Rationale: Typically, breastfeeding is initiated early as soon as possible after birth to promote bonding. The woman may need to use alternate positioning techniques to reduce incisional discomfort. Splinting with pillows helps to reduce the discomfort associated with coughing. Early ambulation is encouraged to prevent respiratory and cardiovascular problems and promote peristalsis. An indwelling urinary catheter is typically inserted to drain the bladder. It usually remains in place for approximately 24 hours

Which intervention would be most appropriate for the nurse to do when assisting parents who have experienced the loss of their preterm newborn? A. Avoid using the terms "death" or "dying." B. Provide opportunities for them to hold the newborn. C. Refrain from initiating conversations with the parents. D. Quickly refocus the parents to a more pleasant topic

Answer: B Rationale: When dealing with grieving parents, nurses should provide them with opportunities to hold the newborn if they desire. In addition, the nurse should provide the parents with as many memories as possible, encouraging them to see, touch, dress, and take pictures of the newborn. These interventions help to validate the parents' sense of loss, relive the experience, and attach significance to the meaning of loss. The nurse should use appropriate terminology, such as "dying," "died," and "death," to help the parents accept the reality of the death. Nurses need to demonstrate empathy and to respect the parents' feelings, responding to them in helpful and supportive ways. Active listening and allowing the parents to vent their frustrations and anger help validate the parents' feelings and facilitate the grieving process.

A set of newborn twins has been admitted to the neonatal intensive care unit with the diagnosis of fetal growth restriction (FGR). Which maternal factors would predispose the newborn to this diagnosis? Select all that apply. A. hemoglobin 15 g/dl (150 g/l) B. A1C levels of 8% (0.08) C. heroin use disorder D. blood pressure baseline of 170/90 mm Hg E. age 39 years F. multiple gestation

Answer: B, C, D, E, F Rationale: Assessment of the small-for-gestational-age (SGA) or FGR infant begins by reviewing the maternal history to identify risk factors such as maternal age over 30 years, a substance use disorder, hypertension, multiple gestation. Gestational diabetes or diabetes mellitus is also a factor. Normal A1C level is 5.7% (0.57) for a person without diabetes. Hemoglobin is normal for pregnant woman in third trimester.

A neonate is exhibiting signs of neonatal abstinence syndrome. Which findings would confirm this diagnosis? Select all that apply. A. adequate rooting and sucking B. frequent sneezing C. persistant fever D. shrill, high-pitched cry E. hypotonic reflexes F. frequent yawning

Answer: B, C, D, F Rationale: Manifestations of neonatal abstinence syndrome include a shrill, high-pitched cry; persistent fever; frequent yawning; and frequent sneezing. Rather than adequate rooting and sucking, these actions will be frantic in a neonate with abstinence syndrome. In addition, these neonates will have hypertonic muscle tone, not hypotonic reflexes.

At the breech forceps birth of a 32 weeks' gestation neonate, the nurse notes olygohydramnios with green thick amniotic fluid. The maternal history reveals a mother of Hispanic ethnicity with marked hypertension, who admits to using cocaine daily. Which factor(s) may contribute to meconium aspiration syndrome (MAS)? Select all that apply. A. the preterm pregnancy B. the forceps breech birth C. maternal cocaine use D. maternal hypertension E. Hispanic ethnicity F. oligohydramnios present

Answer: B, C, D, F Rationale: The predisposing factors for meconium aspiration syndrome include postterm pregnancy and breech presentation with forceps. Ethnicity (Pacific Islander, Indigenous Australian, Black African) is a factor. Postterm neonates are at risk for MAS, but preterm neonates are not. Exposure to drugs during pregnancy, especially tobacco and cocaine, predispose the neeonate to MAS. Maternal hypertension and oligohydramnios also contribute to MAS.

A pregnant woman at 31-weeks' gestation calls the clinic and tells the nurse that she is having contractions sporadically. Which instructions would be most appropriate for the nurse to give the woman? Select all that apply. A. "Walk around the house for the next half hour." B. "Drink two or three glasses of water." C. "Lie down on your back." D. "Try emptying your bladder." E. "Stop what you are doing and rest."

Answer: B, D, E Rationale: Appropriate instructions for the woman who may be experiencing preterm labor include having the client stop what she is doing and rest for an hour, empty her bladder, lie down on her left side, and drink two to three glasses of water.

A couple has just given birth to a baby who has low Apgar scores due to asphyxia from prolonged cord compression. The neonatologist has given a poor prognosis to the newborn, who is not expected to live. Which interventions are appropriate at this time? Select all that apply. A. Advise the parents that the hospital can make the arrangements. B. Offer to pray with the family if appropriate. C. Leave the parents to talk through their next steps. D. Initiate spiritual comfort by calling the hospital clergy, if appropriate. E. Respect variations in the family's spiritual needs and readiness.

Answer: B, D, E Rationale: When assisting the parents to cope with a perinatal loss, the nurse must respect variations in the family's spiritual needs and readiness. The nurse will also initiate spiritual comfort by calling the hospital clergy, if appropriate, and can offer to pray with the family, if appropriate.

A nurse suspects that a client may be developing disseminated intravascular coagulation. The woman has a history of placental abruption (abruptio placentae) during birth. Which finding would help to support the nurse's suspicion? A. severe uterine pain B. board-like abdomen C. appearance of petechiae D. inversion of the uterus

Answer: C Rationale: A complication of abruptio placentae is disseminated intravascular coagulation (DIC), which is manifested by petechiae, ecchymoses, and other signs of impaired clotting. Severe uterine pain, a board-like abdomen, and uterine inversion are not associated with DIC and placental abruption.

. A newborn is suspected of having fetal alcohol syndrome. Which finding would the nurse expect to assess? A. bradypnea B. hydrocephaly C. flattened maxilla D. hypoactivity

Answer: C Rationale: A newborn with fetal alcohol syndrome exhibits characteristic facial features such as microcephaly (not hydrocephaly), small palpebral fissures, and abnormally small eyes, flattened or absent maxilla, epicanthal folds, thin upper lip, and missing vertical groove in the median portion of the upper lip. Bradypnea is not typically associated with fetal alcohol syndrome. Fine and gross motor development is delayed, and the newborn shows poor hand-eye coordination but not hypoactivity.

A nurse is assessing a postterm newborn. Which finding would the nurse correlate with this gestational age variation? A. moist, supple, plum skin appearance B. abundant lanugo and vernix C. thin umbilical cord D. absence of sole creases

Answer: C Rationale: A postterm newborn typically exhibits a thin umbilical cord; dry, cracked, wrinkled skin; limited vernix and lanugo; and creases covering the entire soles of the feet.

The nurse frequently assesses the respiratory status of a preterm newborn based on the understanding that the newborn is at increased risk for respiratory distress syndrome because of: A. inability to clear fluids. B. immature respiratory control center. C. deficiency of surfactant. D. smaller respiratory passages.

Answer: C Rationale: A preterm newborn is at increased risk for respiratory distress syndrome (RDS) because of a surfactant deficiency. Surfactant helps to keep the alveoli open and maintain lung expansion. With a deficiency, the alveoli collapse, predisposing the newborn to RDS. An inability to clear fluids can lead to transient tachypnea. Immature respiratory control centers lead to an increased risk for apnea. Smaller respiratory passages led to an increased risk for obstruction.

. A nurse is developing a plan of care for a woman who is at risk for thromboembolism. Which measure would the nurse include as the most cost-effective method for prevention? A. prophylactic heparin administration B. compression stockings C. early ambulation D. warm compresses

Answer: C Rationale: Although compression stockings and prophylactic heparin administration may be appropriate, the most cost-effective preventive method is early ambulation. It is also the easiest method. Warm compresses are used to treat superficial venous thrombosis.

A nurse is preparing an inservice education program for a group of nurses about dystocia involving problems with the passenger. Which problem would the nurse likely include as the most common? A. macrosomia B. breech presentation C. persistent occiput posterior position D. multifetal pregnancy

Answer: C Rationale: Common problems involving the passenger include occiput posterior position, breech presentation, multifetal pregnancy, excessive size (macrosomia) as it relates to cephalopelvic disproportion (CPD), and structural anomalies. Of these, persistent occiput posterior is the most common malposition, occurring in about 15% of laboring women.

A woman in labor is experiencing hypotonic uterine dysfunction. Assessment reveals no fetopelvic disproportion. Which group of medications would the nurse expect to administer? A. sedatives B. tocolytics C. uterine stimulants D. corticosteroids

Answer: C Rationale: For hypotonic labor, a uterine stimulant such as oxytocin may be prescribed once fetopelvic disproportion is ruled out. Sedatives might be helpful for the woman with hypertonic uterine contractions to promote rest and relaxation. Tocolytics would be ordered to control preterm labor. Corticosteroids may be given to enhance fetal lung maturity for women experiencing preterm labor

A nurse is providing care to a newborn. The nurse suspects that the newborn is developing sepsis based on which assessment finding? A. increased urinary output B. interest in feeding C. temperature instability D. wakefulness

Answer: C Rationale: Manifestations of sepsis are typically nonspecific and may include hypothermia (temperature instability), oliguria or anuria, lack of interest in feeding, and lethargy

The nurse is developing a plan of care for a neonate experiencing symptoms of drug withdrawal. What should be included in this plan? A. Administer glucose between feedings. B. Schedule feedings every 4 to 6 hours. C. Swaddle the infant between feedings. D. Rock horizontally.

Answer: C Rationale: Supportive interventions to promote comfort include swaddling, low lighting, gentle handling, quiet environment with minimal stimulation, use of soft voices, pacifiers to promote "self-soothing," frequent small feedings, and vertical rocking, which will soothe the newborn's neurological system

The parents of a preterm newborn being cared for in the neonatal intensive care unit (NICU) are coming to visit for the first time. The newborn is receiving mechanical ventilation, intravenous fluids and medications and is being monitored electronically by various devices. Which action by the nurse would be most appropriate? A. Suggest that the parents stay for just a few minutes to reduce their anxiety. B. Reassure them that their newborn is progressing well. C. Encourage the parents to touch their preterm newborn. D. Discuss the care they will be giving the newborn upon discharge.

Answer: C Rationale: The NICU environment can be overwhelming. Therefore, the nurse should address their reactions and explain all the equipment being used. On entering the NICU, the nurse should encourage the parents to touch, interact, and hold their newborn. Doing so helps to acquaint the parents with their newborn, promotes self-confidence, and fosters parent-newborn attachment. The parents should be allowed to stay for as long as they feel comfortable. Reassurance, although helpful, may be false reassurance at this time. Discussing discharge care can be done later once the newborn's status improves and plans for discharge are initiated.

. A client is experiencing postpartum hemorrhage, and the nurse begins to massage her fundus. Which action would be most appropriate for the nurse to do when massaging the woman's fundus? A. Place the hands on the sides of the abdomen to grasp the uterus. B. Use an up-and-down motion to massage the uterus. C. Wait until the uterus is firm to express clots. D. Continue massaging the uterus for at least 5 minutes.

Answer: C Rationale: The uterus must be firm before attempts to express clots are made because application of firm pressure on an uncontracted uterus could lead to uterine inversion. One hand is placed on the fundus and the other hand is placed on the area above the symphysis pubis. Circular motions are used for massage. There is no specified amount of time for fundal massage. Uterine tissue responds quickly to touch, so it is important not to overmassage the fundus.

Review of a primiparous woman's labor and birth record reveals a prolonged second stage of labor and extended time in the stirrups. Based on an interpretation of these findings, the nurse would be especially alert for which condition? A. retained placental fragments B. hypertension C. thrombophlebitis D. uterine subinvolution

Answer: C Rationale: The woman is at risk for thrombophlebitis due to the prolonged second stage of labor, necessitating an increased amount of time in bed, and venous pooling that occurs when the woman's legs are in stirrups for a long period of time. These findings are unrelated to retained placental fragments, which would lead to uterine subinvolution, or hypertension.

A woman gave birth to a newborn via vaginal birth with the use of a vacuum extractor. The nurse would be alert for which possible effect in the newborn? A. asphyxia B. clavicular fracture C. cephalhematoma D. central nervous system injury

Answer: C Rationale: Use of forceps or a vacuum extractor poses the risk of tissue trauma, such as ecchymoses, facial and scalp lacerations, facial nerve injury, cephalhematoma, and caput succedaneum. Asphyxia may be related to numerous causes, but it is not associated with use of a vacuum extractor. Clavicular fracture is associated with shoulder dystocia. Central nervous system injury is not associated with the use of a vacuum extractor

Rapid assessment of a newborn indicates the need for resuscitation. The newborn has copious secretiohs. The newborn is dried and placed under a radiant warmer. Which action would the nurse do next? A. Intubate with an appropriate-sized endotracheal tube. B. Give chest compressions at a rate of 80 times per minute. C. Administer epinephrine intravenously. D. Clear the airway with a bulb syring

Answer: D Rationale: After placing the newborn's head in a neutral position, the nurse would clear the airway with a bulb syringe or suction. This is followed by assessment of breathing and bagging if needed, placing a pulse oximeter, ventilating the newborn, assessing the heart rate and giving chest compressions if needed, and then admnistering epinephrine and/or volume expansion if needed

A woman with a history of crack cocaine use disorder is admitted to the labor and birth area. While caring for the client, the nurse notes a sudden onset of fetal bradycardia. Inspection of the abdomen reveals an irregular wall contour. The client also reports acute abdominal pain that is continuous. Which condition would the nurse suspect? A. amniotic fluid embolism B. shoulder dystocia C. uterine rupture D. umbilical cord prolapse

Answer: C Rationale: Uterine rupture is associated with crack cocaine use disorder. Generally, the first and most reliable sign is sudden fetal distress accompanied by acute abdominal pain, vaginal bleeding, hematuria, irregular wall contour, and loss of station in the fetal presenting part. Amniotic fluid embolism often is manifested with a sudden onset of respiratory distress. Shoulder dystocia is noted when continued fetal descent is obstructed after the fetal head is delivered. Umbilical cord prolapse is noted as the protrusion of the cord alongside or ahead of the presenting part of the fetus

. A postpartum woman is prescribed oxytocin to stimulate the uterus to contract. Which action would be most important for the nurse to do? A. Administer the drug as an IV bolus injection. B. Give as a vaginal or rectal suppository. C. Piggyback the IV infusion into a primary line. D. Withhold the drug if the woman is hypertensive

Answer: C Rationale: When giving oxytocin, it should be diluted in a liter of IV solution and the infusion set up to be piggy-backed into a primary line to ensure that the medication can be discontinued readily if hyperstimulation or adverse effects occur. It should never be given as an IV bolus injection. Oxytocin may be given if the woman is hypertensive. Oxytocin is not available as a vaginal or rectal suppository

A nurse is reviewing the medical record of a pregnant client. The nurse suspects that the client may be at risk for dystocia based on which factors? Select all that apply. A. plan for pudendal block anesthetic use B. multiparity C. short maternal stature D. Body mass index 30.2 E. breech fetal presentation

Answer: C, D, E Rationale: Risk factors for dystocia may include maternal short stature, obesity, hydramnios, uterine abnormalities, fetal malpresentation, cephalopelvic disproportion, overstimulation with oxytocin, maternal exhaustion, ineffective pushing, excessive size fetus, poor maternal positioning in labor, and maternal anxiety and fear

A jaundiced neonate must have heel sticks to assess bilirubin levels. Which assessment findings would indicate that the neonate is in pain? Select all that apply. A. There is flaccid muscle tone of the affected limb. B. Respiration rate is 52 breaths per minute. C. Heart rate is 180 beats per minutes. D. Oxygen saturation level is 88%. E. The infant has facial grimacing and quivering chin

Answer: C, D, E Rationale: Suspect pain if the newborn exhibits a sudden high-pitched cry; facial grimace is noted with furrowing of the brow and quivering of the chin with an increase in muscle tone when disturbed. Oxygen desaturation will be noted with an increase in heart rate. Increase in the normal blood pressure, pulse, and respiration are noted.

The nurse is reviewing the physical examination findings for a client who is to undergo labor induction. Which finding would indicate to the nurse that a woman's cervix is ripe in preparation for labor induction? A. posterior position B. firm C. closed D. shortened

Answer: D Rationale: A ripe cervix is shortened, centered (anterior), softened, and partially dilated. An unripe cervix is long, closed, posterior, and firm.

A nurse is teaching a pregnant woman at risk for preterm labor about what to do if she experiences signs and symptoms. The nurse determines that the teaching was successful when the woman makes which statement? A. "I'll sit down to rest for 30 minutes." B. "I'll try to move my bowels." C. "I'll lie down with my legs raised." D. "I'll drink several glasses of water."

Answer: D Rationale: If the woman experiences any signs and symptoms of preterm labor, she should stop what she is doing and rest for 1 hour, empty her bladder, lie down on her side, drink two to three glasses of water, feel her abdomen and note the hardness of the contraction, and call her health care provider and describe the contraction

A thin newborn has a respiratory rate of 80 breaths/min, nasal flaring with sternal retractions, a heart rate of 120 beats/min, temperature of 36° C (96.8° F) and persisting oxygen saturation of <87%. The nurse interprets these findings as: A. cardiac distress. B. respiratory alkalosis. C. bronchial pneumonia. D. respiratory distress.

Answer: D Rationale: Ineffective breathing pattern related to immature respiratory system and respiratory distress as evidenced by tachypnea, nasal flaring, sternal retractions, and/or oxygen saturation less than 87 %. These assessment findings do not indicate bronchial pneumonia respiratory alkalosis or cardiac distress at this time

A nurse is providing education to a woman who is experiencing postpartum hemorrhage and is to receive a uterotonic agent. The nurse determines that additional teaching is needed when the woman identifies which drug as possibly being prescribed as treatment? A. oxytocin B. methylergonovine C. carboprost D. magnesium sulfate

Answer: D Rationale: Magnesium sulfate is during labor as a tocolytic agent to slow or halt preterm labor. It is not be used to treat postpartum hemorrhage. Oxytocin, methylergonovine, and carboprost are drugs used to manage postpartum hemorrhage

A woman who is 2 weeks postpartum calls the clinic and says, "My left breast hurts." After further assessment on the phone, the nurse suspects the woman has mastitis. In addition to pain, the nurse would question the woman about which symptom? A. an inverted nipple on the affected breast B. no breast milk in the affected breast C. an ecchymotic area on the affected breast D. hardening of an area in the affected breast

Answer: D Rationale: Mastitis is characterized by a tender, hot, red, painful area on the affected breast. An inverted nipple is not associated with mastitis. With mastitis, the breast is distended with milk, the area is inflamed (not ecchymotic), and there is breast tenderness.

A newborn has been diagnosed with a group B streptococcal infection shortly after birth. The nurse understands that the newborn most likely acquired this infection from which cause? A. improper hand washing B. contaminated formula C. nonsterile catheter insertion D. mother's birth canal

Answer: D Rationale: Most often, a newborn develops a group B streptococcus infection during the birthing process when the newborn comes into contact with an infected birth canal. Improper hand washing, contaminated formula, and nonsterile catheter insertion would most likely lead to a late-onset infection, which typically occurs in the nursery due to horizontal transmission

A pregnant client undergoing labor induction is receiving an oxytocin infusion. Which finding would require immediate intervention? A. fetal heart rate of 150 beats/minute B. contractions every 2 minutes, lasting 45 seconds C. uterine resting tone of 14 mm Hg D. urine output of 20 mL/hour

Answer: D Rationale: Oxytocin can lead to water intoxication. Therefore, a urine output of 20 mL/hour is below acceptable limits of 30 mL/hour and requires intervention. FHR of 150 beats/minute is within the accepted range of 120 to 160 beats/minute. Contractions should occur every 2 to 3 minutes, lasting 40 to 60 seconds. A uterine resting tone greater than 20 mm Hg would require intervention

As part of an in-service program to a group of home health care nurses who care for postpartum women, a nurse is describing postpartum depression. The nurse determines that the teaching was successful when the group identifies that this condition becomes evident at which time after birth of the newborn? A. in the first week B. within the first 2 weeks C. in approximately 1 month D. within the first 6 weeks

Answer: D Rationale: PPD usually has a gradual onset and becomes evident within the first 6 weeks postpartum. Postpartum blues typically manifests in the first week postpartum. Postpartum psychosis usually appears about 3 months after birth of the newborn.

A neonate born addicted to cocaine is now being treated with medication for acute neonatal abstinence syndrome. Which medication will be prescribed to relieve withdrawal symptoms? A. meperidine B. adrenalin C. naloxone D. morphine sulphate

Answer: D Rationale: Pharmacologic treatment is warranted if conservative measures are not adequate. Common medications used in the management of newborn withdrawal include an opioid (morphine or methadone) and phenobarbital as a second drug if the opiate does not adequately control symptoms. The other drugs are not used in NAS treatment.

The nurse is conducting a class for postpartum women about mood disorders. The nurse describes a transient, self-limiting mood disorder that affects mothers after birth. The nurse determines that the women understood the description when they identify the condition as postpartum: A. depression. B. psychosis. C. bipolar disorder. D. blues.

Answer: D Rationale: Postpartum blues are manifested by mild depressive symptoms of anxiety, irritability, mood swings, tearfulness, increased sensitivity, feelings of being overwhelmed, and fatigue. They are usually self-limiting and require no formal treatment other than reassurance and validation of the woman's experience as well as assistance in caring for herself and her newborn. Postpartum depression is a major depressive episode associated with birth. Postpartum psychosis is at the severe end of the continuum of postpartum emotional disorders. Bipolar disorder refers to a mood disorder typically involving episodes of depression and mania

After spontaneous rupture of membranes, the nurse notices a prolapsed cord. The nurse immediately places the woman in which position? A. supine B. side-lying C. sitting D. knee-chest

Answer: D Rationale: Pressure on the cord needs to be relieved. Therefore, the nurse would position the woman in a modified Sims, Trendelenburg, or knee-chest position. Supine, side-lying, or sitting would not provide relief of cord compression

The nurse is teaching a group of parents who have preterm newborns about the differences between a full-term newborn and a preterm newborn. Which characteristic would the nurse describe as associated with a preterm newborn but not a term newborn? A. fewer visible blood vessels through the skin B. more subcutaneous fat in the neck and abdomen C. well-developed flexor muscles in the extremities D. greater body surface area in proportion to weight

Answer: D Rationale: Preterm newborns have large body surface areas compared to weight, which allows an increased transfer of heat from their bodies to the environment. Preterm newborns often have thin transparent skin with numerous visible veins, minimal subcutaneous fat, and poor muscle tone.

. A nurse is assessing a postpartum client. Which finding would the cause the nurse the greatest concern? A. leg pain on ambulation with mild ankle edema B. calf pain with dorsiflexion of the foot C. perineal pain with swelling along the episiotomy D. sharp, stabbing chest pain with shortness of breath

Answer: D Rationale: Sharp, stabbing chest pain with shortness of breath suggests pulmonary embolism, an emergency that requires immediate action. Leg pain on ambulation with mild edema suggests superficial venous thrombosis. Calf pain on dorsiflexion of the foot may indicate deep vein thrombosis or a strained muscle or contusion. Perineal pain with swelling along the episiotomy might be a normal finding or suggest an infection. Of the conditions, pulmonary embolism is the most urgent.

A woman who is experiencing postpartum hemorrhage is extremely apprehensive and diaphoretic. The woman's extremities are cool and her capillary refill time is increased. Based on this assessment, the nurse suspects that the client is experiencing approximately how much blood loss? A. 20% B. 30% C. 40% D. 60%

Answer: D Rationale: The client's assessment indicated mild shock, which is associated with a 20% blood loss. Moderate shock occurs with a blood loss of 30 to 40%. Severe shock is associated with a blood loss greater than 40%.

A preterm newborn is receiving enteral feedings. Which finding would alert the nurse to suspect that the newborn is developing NEC? A. irritability B. sunken abdomen C. clay-colored stools D. feeding intolerance

Answer: D Rationale: The newborn with NEC may exhibit feeding intolerance with lethargy, abdominal distention and tenderness, and bloody stools.

A nurse is assessing a preterm newborn. Which finding would alert the nurse to suspect that a preterm newborn is in pain? A. bradycardia B. oxygen saturation level of 94% C. decreased muscle tone D. sudden high-pitched cry

Answer: D Rationale: The nurse should suspect pain if the newborn exhibits a sudden high-pitched cry, oxygen desaturation, tachycardia, and increased muscle tone

The nurse is admitting a term, large-for-gestational-age neonate weighing 4,610 g (10 lb, 2 oz), born vaginally with a mid-forceps assist, to a 15-year-old primipara. What would the nurse anticipate as a result of the birth? A. fracture of the tibia B. fracture of the femur C. fracture of a rib D. midclavicular fracture

Answer: D Rationale: Trauma to the newborn may result from the use of mechanical forces, such as forceps during birth. Primarily injuries are found in large babies and babies with shoulder dystocia. Associated traumatic injuries include fracture of the clavicle or humerus or subluxations of the shoulder or cervical spine.

A neonate is diagnosed with Erb's palsy after birth. The parents are concerned about their neonate's limp arm. The nurse explains the neonate will be scheduled to receive what recommended treatment for this condition first? A. Physical therapy to the joint and extremity B. Nothing but time and let nature take its course C. Surgery to correct the joint and muscle alignment D. Immobilization of the shoulder and arm

Answer: D Rationale: Treatment for a neonate with Erb palsy usually involves immobilization of the upper arm across the upper abdomen/chest to protect the shoulder from excessive motion for the first week; then gentle passive range-of-motion exercises are performed daily to prevent contractures. Surgery is not needed to regain function since there is no structural injury. Doing nothing will not help the neonate regain function in the extremity.


संबंधित स्टडी सेट्स

Prep U: Chapter 38 Oxygenation and Perfusion

View Set

lower respiratory disorders- med surg

View Set

IB biology super review for paper 1 + 2

View Set

PSY 2 Chapter 26, 25, 17, 13, 35

View Set

AP US History Vocabulary (Presidential Highlights and Study Guides)

View Set

Chapter 14: Care of a patient with a Neurologic disorder exam

View Set

Writing a Persuasive E-mail Assignment

View Set

The Lean StartUp, Identify Your Niche, Finding Your Audience Quiz, Finding Your Passion

View Set